Section 10 Flashcards

1
Q

A 60-year-old woman presents to the office following a thalamic stroke nine months ago. Since her stroke, she complains of diffuse burning pain that is worse in her extremities. She reports significant fatigue and sensitivity to temperature. She denies loss of muscle strength. She has been treated with amitriptyline without relief. Her physical exam is significant for hypersensitivity to light palpation of her skin. What is the best next step in the management of this patient?

A
  1. Continue amitriptyline and add tramadol
    2. Continue amitriptyline and add gabapentin
  2. Discontinue amitriptyline then perform motor cortex stimulation
  3. Discontinue amitriptyline then perform brain stimulation

  • This patient has central pain syndrome. There is strong evidence for the use of tricyclic antidepressants such as amitriptyline, SNRIs such as duloxetine or venlafaxine, and the anticonvulsants pregabalin and gabapentin. There is moderate evidence for the use of tramadol or an SSRI and weak evidence for using S-adenosyl-L-methionine (SAMe).
  • Gabapentin can alleviate chronic pain in select patients with centralized pain, particularly those with peripheral neuropathy.
  • According to a 2015 review of the effects of gabapentin on central stroke-related pain, over 50% of patients in the study had clinically significant pain improvement with gabapentin.
  • Motor cortex stimulation (MCS) and deep brain stimulation (DBS) are effective treatment modalities for patients with refractory pain, centralized pain, and peripheral neuropathy.
How well did you know this?
1
Not at all
2
3
4
5
Perfectly
2
Q

A 35-year-old female patient presents to the clinic with complaints of irregular menstrual cycles and a thick yellowish fluid oozing out of her breasts. She is sexually active and has been married for four years, but for no known reason, she is unable to get pregnant. The rest of the physical examination is unremarkable. The thyroid function tests are within a normal range, and the serum prolactin level is 1200 ng/ml (4.8 to 23.3 ng/ml). MRI shows a 2.0 cm pituitary mass. Which of the following drugs is used to treat this pathology?

A

1. Cabergoline
2. Synthetic thyroid hormone
3. Selective serotonin reuptake inhibitors
4. Oral contraceptives

  • Treatment of hyperprolactinemia depends on the cause. Once the physiological cause of hyperprolactinemia is excluded, one should look for other possible systemic causes and address them for symptomatic patients.
  • Treatment is only necessary if patients develop hypogonadism, osteoporosis, or troublesome galactorrhea. If hyperprolactinemia is suspected because of drugs, it should be discontinued temporarily, if possible, to see if the prolactin level normalizes. If medication cannot be discontinued, especially antipsychotics, it should be changed to a different antipsychotic agent that does not increase prolactin, e.g., quetiapine. If that is not possible, the addition of dopamine agonists should be considered. These changes should be made in consultation with a psychiatrist. The addition of estradiol in women and testosterone in men can be considered for hypogonadism and low bone mass. If the prolactin level does not normalize after stopping medication or if discontinuation of medication is not possible, pituitary MRI should be done.
  • Endocrine society guideline recommends against treatment with a dopamine agonist for asymptomatic microadenoma but recommends dopamine agonist therapy to decrease prolactin levels and tumor size and normalize gonadal function for symptomatic patients with microadenomas or macroadenomas. Cabergoline is preferred to other dopamine agonists due to its higher efficacy in normalizing prolactin levels and tumor shrinkage.
  • Most prolactinomas are treated with medical therapy only. Surgery and radiotherapy are reserved for those who are resistant to medical therapy with dopamine agonists. Endoscopic endonasal transsphenoidal surgery is a preferred surgical method. Prophylactic surgery is considered in women with large prolactinomas, which potentially threatens vision during pregnancy.
How well did you know this?
1
Not at all
2
3
4
5
Perfectly
3
Q

A 15-year-old boy is brought to the emergency department with complaints of progressive headache and multiple episodes of projectile vomiting 5 months after the revision of a hydrocephalus shunt. The patient has optic disc edema. The shunt chamber does not recoil following its manual compression. There is poor CSF flow during a diagnostic shunt tap. Which of the following is the most likely etiology for the clinical deterioration observed in this patient?

A
  1. Distal shunt failure
    2. Proximal catheter obstruction
  2. Meningitis
  3. Peritonitis

  • The patient has features of raised intracranial pressure such as headache, vomiting, and optic disc edema. Shunt obstruction is a common complication associated with shunt placement.
  • There is evidence of proximal shunt failure resulting in its obstruction since there is no recoil of the chamber following its manual compression.
  • The poor CSF flow during diagnostic shunt tap has more than 90 percent positive predictive value in diagnosing the condition.
  • The distal catheter obstruction will show difficulty in compressing the valve following its recoil and occluding the proximal catheter. Shunt infection and meningitis present with high-grade fever, nuchal rigidity, and altered sensorium. In the case of peritonitis, there will be features of rebound abdominal tenderness.
How well did you know this?
1
Not at all
2
3
4
5
Perfectly
4
Q

The nurse mishears the order for “2 and 50” and instead gives 250 mcg of fentanyl. Two minutes later the oxygen saturation is 80 percent. What should be administered first?

A
  1. 0.4 mg flumazenil
  2. 0.4 mg naloxone
  3. 0.5 mg atropine
    4. Oxygen

  • This patient is experiencing opioid-induced respiratory depression from a large fentanyl dose.
  • While reversal with naloxone may be required, oxygen should be administered first.
  • Along with oxygen administration, attempts to arouse the patient and open the airway should be performed.
  • Failing these maneuvers, reversal may be required.
How well did you know this?
1
Not at all
2
3
4
5
Perfectly
5
Q

A 45-year-old man patient presents to the clinic following progressive headache and diminution of vision. Visual field study reveals the presence of bitemporal hemianopia in the patient. MRI reveals giant pituitary macroadenoma with suprasellar extension. An endoscopic endonasal trans-sellar excision of the tumor is planned. Intraoperatively, while trying to remove the tumor on the cavernous wall, there is a sudden jet of blood into the operating field. The operating surgeon controlled the bleeding with rigorous packing with hemostatic agents into the cavernous wall. The remaining portion of the tumor is removed. In the postoperative period, the patient complains of severe headaches. The patient also develops ptosis with ophthalmoplegia. He further develops progressive painful proptosis. Which of the following is the next best step in the management of this patient?

A
  1. MR angiography
  2. CT angiography
  3. Stereotactic radiation
    4. Digital subtraction angiography (DSA)

  • The patient has developed vascular cavernous sinus syndrome owing to the injury to the cavernous portion of the internal carotid artery.
  • The characteristic syndrome in the patient is most likely following either rupture of the pseudoaneurysm or the development of the direct carotid-cavernous fistula in the patient.
  • The digital subtraction angiography is diagnostic as well as therapeutic in the patient. The embolization of the pseudoaneurysm or the placement of the pipeline flow diverters can be undertaken in the case of carotid-cavernous fistula.
  • The MR or CT angiography is only of diagnostic value. The stereotactic radiation is justified in limited cases of such vascular complications, especially in a patient who fails endovascular procedure. The clinical benefits are seen after a considerable period of time only, and the patient also has a high risk of recurrence.
How well did you know this?
1
Not at all
2
3
4
5
Perfectly
6
Q

A 45-year-old female is seen in the outpatient clinic due to the incidental finding of a 1.7 cm x 1.9 cm isodense left lateral ventricle mass seen on the head computed tomographic scan with contrast done for workup for chronic sinusitis. On magnetic resonance imaging, the mass is located adjacent to the choroid plexus of the atrium of the left lateral ventricle and is isointense to gray matter on T1/T2 and has homogenous/vividly enhancing on post-contrast images. Also, there is evidence of fine calcified borders on the gradient echo sequence. No evidence of hydrocephalus or parenchymal edema is seen. On the physical exam, the patient is neurologically intact and only complains of a bifrontal headache. What is the best next step in management?

A
  1. Consult neurosurgery for an urgent operation
    2. Observation
  2. Radio-oncologist consult for elective radiosurgery
  3. Obtain serum beta-human chorionic gonadotropin, alpha- fetoprotein, and placental alkaline phosphatase

  • Meningiomas are usually isointense to grey matter (60-90%) on T1 and T2 sequences. Intraventricular meningiomas are mainly benign, slow-growing lesions. However, it is important to follow- up with serial imaging due to the risk of ventricular trapping and symptomatic parenchymal edema that may warrant surgical intervention.
  • Meningiomas appear more common in the atrium on the left lateral ventricle and are well-circumscribed, homogeneously enhancing masses.
  • Compared to extra-axial meningiomas, intraventricular meningiomas have a higher percentage of calcification (50 vs. 20%).
  • Due to the absence of hydrocephalus, choroid plexus papilloma is unlikely. Due to the absence of adjacent brain parenchymal edema, choroid plexus carcinoma and brain metastasis are unlikely. There is no need for urgent surgery or radiosurgery.
How well did you know this?
1
Not at all
2
3
4
5
Perfectly
7
Q

A 17-year-old female, with no comorbidities, is brought to the emergency department with a history of a motor vehicle accident involving a car (which she was driving) and a bus, one hour back. On examination, her pulse rate is 80/minute, blood pressure 130/80 mmHg, respiratory rate 16/minute, and oxygen saturation is 98%. Her airway is clear, and air entry is bilaterally equal in all lung fields. Her Glasgow Coma Scale (GCS) score is E3V4M6, and her pupils are bilaterally 3 mm in size and react to light. A chest X-ray taken shows clear lung parenchyma. A computed tomogram scan of the brain is taken, which shows an undisplaced right frontal bone fracture and an underlying contusion of 15 ml volume. There is no midline shift or mass effect. She is admitted to the neurosurgery intensive care unit. Two hours later, the patient develops tachypnea (respiratory rate of 30/minute), while GCS score and pupils remain the same. Within 2 minutes, her saturation drops down to 80%, and she has to be intubated and ventilated. On chest auscultation, there are bilateral basal crackles. A chest X-ray shows bilateral hyperdense infiltrates in the lung parenchyma. Which part of the brainstem is likely to contribute to the current deterioration in her clinical condition?

A
  1. The dorsolateral aspect of the pons
  2. The ventrolateral aspect of the pons
  3. The dorsolateral aspect of the medulla
    4. The ventrolateral aspect of the medulla

  • The patient has neurogenic pulmonary edema. Rapid developments characterize the early stages of neurogenic pulmonary edema. The patients are usually children or young adults who have suffered an intracranial injury recently.
  • The clinical signs boil down to classic signs of pulmonary edema with the absence of signs of left ventricular failure usually found in cardiogenic edema.
  • Although for classic neurogenic pulmonary edema, the manifestation could be detected clinically within 2 to 12 hours post-injury; presentation may take days in some cases.
  • Area A1, situated in the ventrolateral aspect of the medulla, is one of the main trigger centers for neurogenic pulmonary edema. It is composed of catecholamine neurons that project to the hypothalamus.
How well did you know this?
1
Not at all
2
3
4
5
Perfectly
8
Q

A 65-year-old male with a past medical history of atrial fibrillation on warfarin and metoprolol, gastroesophageal disease, and type 2 diabetes on metformin presents to the emergency department after the onset of word slurring and left facial droop 3 hours ago. He is still unable to articulate a complete sentence. After a comprehensive neurologic exam and imaging, an area of decreased perfusion is found in the territory supplied by the middle cerebral artery, and tPA is administered. The patient’s symptoms partially improve. Several hours after treatment, the patient develops worsening confusion and arm weakness. Repeat computed tomography scan shows a new large area of increased attenuation in the lateral parietal lobe. What is the appropriate next step?

A
  1. Repeat tPA
  2. Computed tomography scan with IV contrast
    3. Neurosurgical intervention
  3. Magnetic resonance angiogram

  • Post-tPA intracranial hemorrhage should be considered if the patient develops a worsening neurologic exam, new headache, change in the level of consciousness, increased weakness, or sudden change in blood pressure. Risk factors include older age, greater stroke severity, hyperglycemia/diabetes, hypertension, atrial fibrillation, heart disease, and anticoagulant use.
  • This pathology usually happens within 36 hours after thrombolytic infusion. Only about half of the patients with this condition are diagnosed 5–10 hours after the administration of tPA.
  • The first step taken should be to immediately stop the infusion of tPA if it is still running. Computed tomography scan of the brain is the first imaging step in the evaluation of possible stroke or when there is suspicion of conversion to hemorrhagic after administration of thrombolytic agents.
  • If hemorrhage is found (seen on computed tomography scan as the previous ischemic infarct with superimposed hyperintense cerebral hemorrhage), the next steps would be to collect baseline labs and reverse the anticoagulation using cryoprecipitate or other products. In addition, neurosurgery should be called for potential surgical treatment.
How well did you know this?
1
Not at all
2
3
4
5
Perfectly
9
Q

A 60-year-old male is being evaluated at the emergency department for a frontal right-sided headache. Skull X- rays show mild opacification of the right frontal sinus. Brain magnetic resonance imaging with and without contrast is performed. It shows an incidental 2.0 cm x 1.8 cm homogeneous enhancing mass at the atrium of the left lateral ventricle without evidence of ventricular trapping or edema on adjacent brain parenchyma. The mucosa of the frontal sinus is thickened and enhanced. On the physical exam, the patient is neurologically intact. He is prescribed nasal decongestants. What is the best next step in management for the incidental mass?

A

1. Follow-up brain magnetic resonance imaging with and without contrast in 6 months
2. Stereotactic needle biopsy
3. Left-sided craniotomy for tumor resection
4. Radio-oncologist consult for elective radiosurgery

  • A well-circumscribed, homogeneously enhancing mass in the atrium of the lateral ventricle is most likely an intraventricular meningioma.
  • Intraventricular meningiomas are mainly benign, slow-growing lesions. However, it is essential to follow-up with serial imaging due to the risk of ventricular trapping and symptomatic parenchymal edema that may warrant surgical intervention.
  • If there is documentation of tumor growth on follow-up magnetic resonance imaging and the diagnosis of meningioma by brain magnetic resonance imaging is clear. One may proceed with treatment with radiosurgery if the lesion is under 3.0 cm in all dimensions. If the diagnosis is unclear, then a less invasive stereotactic needle biopsy vs. open tumor resection is strongly suggested.
  • Even if meningioma is found incidentally, when there is evidence of associated parenchymal edema or trapped lateral ventricle, then urgent/emergent surgery should be performed.
How well did you know this?
1
Not at all
2
3
4
5
Perfectly
10
Q

A 71-year-old male smoker with a past medical history of right lung adenocarcinoma presents with a small pupil on the right, drooping of the right eyelid, and decreased sweating on the right side of his face. Which of the following conditions would increase the size discrepancy between the pupils?

A
  1. Examination in a brightly lit room
    2. Examination in a dark room
  2. Have the patient focus on a near object
  3. Pharmacologically constrict the pupil prior to testing

  • This patient likely has Horner syndrome from disruption of the sympathetic chain along the apex of the lung due to his malignancy. Horner’s syndrome causes miosis, ptosis, and anhydrosis.
  • The miotic pupil cannot dilate well. Therefore, examining the patient in a relatively dark room will increase the discrepancy in size between the pupils, since the other pupil will more effectively dilate.
  • The patient should focus on a distant object to eliminate the near reflex. Pharmacologically constrict the pupil prior to testing would not be helpful.
  • A bright room makes the evaluation of pupillary response to light difficult.
How well did you know this?
1
Not at all
2
3
4
5
Perfectly
11
Q

A 52-year-old female presented to the clinic with a 4-month history of pain and burning in the left hemithorax, initially treated as symptoms of herpes, without a satisfactory result, until it became more acute 13 days ago. Her past medical history is significant for type 2 diabetes mellitus for the past 4 years. She is taking metformin and sitagliptin for diabetes. Her BMI is 36 kg/m2. She underwent laparoscopic cholecystectomy 8 years ago. On physical exam, she has mild paraspinal tenderness, normal patellar reflexes, normal muscle strength in her lower extremities, and a normal gait exam. MRI thoracolumbar spine reveals thoracic disk herniation at T7-T8 interspace. Which of the following is the next best step in the management of this patient?

A
  1. Thoracic laminectomy
    2. Thoracic discectomy
  2. Physical therapy and NSAIDs 4. Epidural steroids

  • The clinical presentation and imaging studies are consistent with a thoracic disk herniation with spinal cord compression causing symptoms of thoracic myelopathy.
  • Surgical intervention is considered the treatment of choice for symptomatic thoracic disc herniations.
  • Surgery will allow for the removal of the ossified disc, decompressing the region, and relieving pressure on the nerve or spinal cord.
  • The initial treatment of thoracic discogenic syndrome is usually conservative (nonoperational) since some disc herniations have been reported to stabilize/regress with time, especially in younger patients. However, this patient is displaying signs and symptoms of thoracic myelopathy, which warrants surgical management.
How well did you know this?
1
Not at all
2
3
4
5
Perfectly
11
Q

A 35-year-old man presents with a two-month history of a gradually enlarging, painful scrotal mass. He has a history of acromegaly, for which he underwent transsphenoidal surgery, thyroid nodules, and multiple lentigines. The patient was adopted and does not take any medications. Examination is notable for diffuse blue-black ovoid macules and a firm 4 cm scrotal mass. After further workup consistent with a Leydig cell tumor, the patient undergoes left radical orchiectomy and subsequently is found to have a cardiac myxoma. Given the likely diagnosis, which of the following manifestations of this patient’s condition is most associated with mortality?

A
  1. Acromegaly
    2. Cardiac disease
  2. Skin malignancy
  3. Testicular cancer

  • Patients with Carney complex have mortality most associated with cardiac disease, specifically cardiac myxomas.
  • Cardiac myxomas are associated with emboli and cardiac deficiency, for which even surgical excision can be insufficient as they can recur.
  • Cardiac tumors can cause death in patients with Carney complex, either directly related to the tumors or surgical complications that occur during or after their removal.
  • Other major causes of mortality include metastatic or intracranial psammomatous melanotic schwannoma, carcinoma or metastatic tumor, and non-cardiac postoperative complications
How well did you know this?
1
Not at all
2
3
4
5
Perfectly
12
Q

A 4-year-old boy is brought to the provider with an inability to carry out his daily activities. He has difficulty running, climbing stairs, and falls frequently. His maternal uncle, who died of respiratory failure, had similar problems and was wheelchair-bound by the age of 12 years. What is the diagnostic test for this patient?

A
  1. Creatine kinase levels
  2. Whole-body PET scan
  3. Electromyography
    4. Gene analysis

  • Duchenne muscular dystrophy (DMD) is a severe form of inherited muscular dystrophies. It is described as the most common hereditary neuromuscular disease that does not have a predilection for any race or ethnic group. Mutations in the dystrophin gene cause progressive muscle fiber degeneration and weakness.
  • This type of dystrophy initially may present with difficult mobility but progresses to such an extent that patients are unable to carry out activities of daily living and must use wheelchairs. Cardiac and orthopedic complications are not uncommon, and death usually occurs in the twenties due to respiratory muscle weakness or cardiomyopathies.
  • A dystrophinopathy should be considered in patients with symptoms of weakness, a characteristic physical exam, and a possible family history of the disease.
  • Patients with DMD exhibit the complete or near-complete absence of the dystrophin gene. This is a diagnostic test of DMD. Dystrophin immunoblotting can be utilized to predict the severity of the disease.
How well did you know this?
1
Not at all
2
3
4
5
Perfectly
12
Q

A 62-year-old male has a one-week history of nausea, vomiting, and paresthesias of the right hand and foot. On the day of admission, he had a generalized seizure followed by a headache that increases when he bends forward. There is right facial droop and loss of double simultaneous tactile stimulation. MRI shows a left posterior frontal, intraparenchymal, non-enhancing mass with a local mass-effect on the lateral ventricle consistent with a brain tumor. Transformation of which brain cell type is most likely the etiology of the tumor in this patient?

A
  1. Lymphocyte
  2. Neuron
    3. Star-shaped glial cells
  3. Oligodendrocyte

  • The most common primary brain tumors in adults are in the family of astrocytomas. Within the astrocytomas. Glioblastoma Multiforme, WHO grade 4 is most common with a peak incidence in ages 60-70 with a male predominance.
  • Astrocytoma originates in astrocytes, which are a kind of glial cells in the cerebrum which are star-shaped. It is the most common glioma, usually affecting the brain and sometimes the spinal cord. Amongst brain tumors, glial tumors comprise 60% of the tumors.
  • Symptoms can be divided into two categories: general & focal. General symptoms include headache (usually early morning), nausea, vomiting, cognitive difficulties, personality changes, and gait disorders. Localizing symptoms include seizures, aphasia, or visual field defects. A visual field defect is commonly unnoticed by the patient, may be revealed after it leads to injury such as automobile accidents.
  • The only test necessary to diagnose a brain tumor is neuroimaging. MRI is the best imaging for the same. Gadolinium contrast-enhanced MR imaging should be used whenever possible. If there is any contraindication for MRI, such as joint implants or pacemakers in situ, computed tomography or CT may be done. Lower-grade gliomas aren’t contrasting enhancing, so fluid-attenuated inversion recovery (FLAIR) sequences of MRI are done. If a tumor is found, a biopsy must be performed.
How well did you know this?
1
Not at all
2
3
4
5
Perfectly
13
Q

A 28-year-old man presents for preoperative evaluation before elective surgery for a distal ulnar fracture. He injured his arm during a basketball game and has been in a sling since the incident. He reports numbness and tingling. When recording somatosensory evoked potentials (SEPs) from the patient’s injured arm, which potential is most likely to show changes from baseline?

A

1. N9
2. N14
3. N20
4. N22

  • Somatosensory evoked potentials (SEP) monitoring is employed intraoperatively to identify, and ideally, avoid impending permanent damage to neurologic structures. SEP waveforms change in response to many different factors, some of which can cause permanent damage.
  • The N9 potential is generated at Erb’s point and reflects the peripheral components of the monitored sensory pathway.
  • The mechanism of the patient’s injury is most consistent with a peripheral nerve injury, which would affect the N9 potential.
  • The N22 potential is generated at the gray matter of the spinal cord. It is unlikely this potential would be changed, given the patient’s reported mechanism of injury. The N20 potential is generated at the cortex and would not show abnormalities due to the patient’s fall. The N14 potential is generated at the cervicomedullary area.
How well did you know this?
1
Not at all
2
3
4
5
Perfectly
13
Q

A 34-year-old woman with a BMI of 42 kg/m2 presents to the clinic with complaints of numbness and paresthesias in the lateral aspect of the right thigh. The patient says she has tried gabapentin in the past with no relief in symptoms. She had an MRI of the lumbar spine, which was reported normal. What is the most appropriate placement of the peripheral stimulator lead in this patient?

A

1. Lateral femoral cutaneous nerve
2. L2 nerve root
3. Sciatic nerve
4. Femoral nerve

  • The patient’s clinical symptoms are consistent with meralgia paresthetica or lateral femoral cutaneous nerve syndrome.
  • This condition is most common in morbidly obese females due to chronic femoral cutaneous nerve compression with tight clothing.
  • Peripheral nerve stimulation is an efficacious treatment option for this condition where the PNS leads are placed parallel to the lateral femoral cutaneous nerve.
  • All the other nerves do not match the area of distribution of paresthesia.
How well did you know this?
1
Not at all
2
3
4
5
Perfectly
14
Q

A 50-year-old man presents to the clinic with left- sided facial pain for the past 6 months. He describes it as a sharp stabbing pain along the left forehead, and it appears in short episodes. During the episodes, he experiences lacrimation. He uses carbamazepine but is seeking a permanent solution. Radiofrequency thermocoagulation of the implicated ganglion is planned. What is the most appropriate anatomic location to mark the site for needle entry for this procedure in this patient?

A

1. Anterior to the mandible and inferior to the zygomatic arch
2. Medial to the occipital artery
3. Directly on the point of maximal tenderness
4. At the infraorbital foramen

  • This patient most likely has trigeminal neuralgia due to episodes of sharp unilateral pain in the V1 territory along with lacrimation.
  • The infrazygomatic approach to sphenopalatine ganglion radiofrequency ablation involves entering the skin anterior to the mandible and inferior to the zygomatic arch.
  • Sphenopalatine ganglion RFA is a relatively quick, cost- effective, and safe procedure with a low complication rate. It has demonstrated that it is a useful method to treat chronic cluster headaches.
  • The location for an occipital nerve block is medial to the occipital artery. Injecting directly on the point of maximal tenderness is a technique for trigger point injections and would not be indicated for a patient experiencing trigeminal neuralgia. The infraorbital foramen is the location for an infraorbital nerve block. This would be effective for pain involving the lower eyelid, nares, and upper lip. It would not be effective for pain on the forehead due to trigeminal neuralgia.
How well did you know this?
1
Not at all
2
3
4
5
Perfectly
14
Q

A 35-year-old patient presents with acute onset worst headache of his life. CT head revealed an acute subarachnoid hemorrhage. CT cerebral angiography showed evidence of an ophthalmic artery aneurysm. The treating clinician planned for microsurgical clipping of the aneurysm. He however wants to safeguard from the possible intraoperative rupture of the lesion. Which of the following is the most rational approach to control hemorrhage following the same in the patient?

A
  1. Proximal temporary clip application
  2. Suture control of the proximal internal carotid artery
  3. Adenosine induced cardiac arrest
    4. Neck control of the internal carotid artery

  • Inadvertent rupture of the ophthalmic artery aneurysm is a devastating complication with great difficulty in controlling the resulting hemorrhage.
  • Preoperative neck control of the internal carotid artery is the most rational approach to control such adverse intraoperative events.
  • This effectively minimizes the blood inflow in the internal carotid thereby allowing a surgical environment in the correct and safe application of the permanent clipping.
  • An adenosine-induced cardiac arrest requires expert neuro anesthesia support. Application of the proximal temporary clip is not feasible in the case of an ophthalmic segment of the internal carotid artery.
How well did you know this?
1
Not at all
2
3
4
5
Perfectly
15
Q

A 17-year-old male patient with severe neurological injury after a motor vehicle collision requires prolonged mechanical ventilation. He has been intubated for 14 days and remains at a Glasgow coma scale of 6 since his decompressive craniotomy two weeks ago. He has a nasogastric tube in place and has been on stable ventilator settings for the past 48 hours but has failed ventilator weaning multiple times. He initially developed post- traumatic diabetes insipidus after his accident, but his fluid balance has been stable for five days. He is being prepared to be transferred to a neurological care center, which requires a tracheostomy before transportation there. What is the next best step in management?

A
  1. Percutaneous gastrostomy to prevent tracheoesophageal fistula
  2. Electroencephalogram to determine higher brain function to aid in prognosis discussion
    3. Discussion regarding goals of care with the family and caregivers
  3. Placement of a low-pressure cuffed tracheostomy tube to minimize the risk of subglottic or tracheal stenosis

  • This patient has suffered a devastating neurological injury, as evidenced by his Glasgow coma score of 6. This portends a dismal prognosis as it has been unchanged for 14 days.
  • It is important to ascertain the patient’s and the family’s wishes and goals of care before embarking upon additional procedures in tragic situations such as this, and palliative and hospice options must be discussed.
  • The respiratory drive is in the reptilian brain and may persist even if higher cortical functions have been lost. This can help families interpret what their loved ones would have wanted in such a situation.
  • Electroencephalogram (EEG) can be used to assess higher cortical electrical activity and may aid in such discussions. This is required in some U.S. states but can provide another prognostic indicator to provide to families regarding the potential recovery.
How well did you know this?
1
Not at all
2
3
4
5
Perfectly
15
Q

A 24-year-old woman falls approximately 15 feet from a ladder while cleaning her gutters. At the scene, she is non- responsive and intubated. She is taken to the local hospital, where imaging shows diffuse axonal injury with numerous microhemorrhages and shear injury. The patient is given multiple medications to manage presumed intracranial hypertension from cerebral edema, and an intracranial pressure monitor is placed. Her intracranial pressure is initially controlled with medical interventions, including intravenous mannitol, but at 36 hours post-injury, her intracranial pressure begins to rise and worsens despite medical intervention. What is the mechanism most likely responsible for her increased intracranial pressure?

A
  1. Dehydration
    2. Medication extravasation
  2. Pyretic effects of medications
  3. Upregulated metabolism of medications

  • Mannitol is an osmotic diuretic used to treat intracranial hypertension and cerebral edema.
  • With frequently repeated doses of mannitol, the mannitol can begin to leach/extravasate across a leaky blood-brain barrier and effectively draw water into the brain, causing worsening cerebral edema.
  • Doses of mannitol range from 0.25 g/kg to 2 g/kg given over 30- 60 minutes, with effects lasting approximately 6 hours.
  • Mannitol is contraindicated in patients with severe renal disease, severe pulmonary disease, dehydration, progressive heart failure, or electrolyte abnormalities.
How well did you know this?
1
Not at all
2
3
4
5
Perfectly
16
Q

A 28-year-old male is shifted to the intensive care unit due to an altered sensorium associated with a high-grade fever. He was diagnosed with acute myeloid leukemia and had undergone 2 cycles of induction chemotherapy with the completion of the last cycle 10 days ago. On the examination, there is erythema and black eschar surrounding the left eye. The left pupil was fixed & dilated. CT revealed enhancing mass in the ethmoidal sinus as well. The periodic acid-Schiff stain of the material obtained following the surgical debridement demonstrated broad hyphae with right-angle branching. What is the best next step in the management of the patent?

A

1. Amphotericin B
2. Fluconazole
3. Antitubercular therapy
4. Chloroquine

  • The clinical signs and symptoms in the patent are highly suggestive of a rhino-orbital mucormycosis.
  • The periodic acid-Schiff stain demonstrating broad hyphae with right-angle branching is the hallmark of the disease process.
  • Parenteral Amphotericin B is advocated for the management of rhino-orbital mucormycosis.
  • Liposomal amphotericin has better tissue penetration and minimal renal toxicities, however, the cost is a major limiting factor.
How well did you know this?
1
Not at all
2
3
4
5
Perfectly
17
Q

A 6-year-old boy is diagnosed with a posterior fossa cystic tumor and is awaiting surgery. The neurosurgeon informs the patient’s parents that he could develop acute hydrocephalus if there is bleeding into the tumor. Which of the following would be the last sign to manifest if the patient develops this complication?

A
  1. Headache
    2. Symptomatic papilledema
  2. Abducens nerve palsy
  3. Ataxia

  • Critically increased intracranial pressure initially will cause decreased consciousness.
  • Other signs and symptoms are likely to occur in the following sequence: headache, nausea, vomiting, irritability, and abducens nerve palsy.
  • Papilledema is not present in all patients with increased intracranial pressure.
  • Symptomatic papilledema with vision loss only occurs with chronic or advanced increased intracranial pressure after the initial sequelae of
How well did you know this?
1
Not at all
2
3
4
5
Perfectly
17
Q

A previously healthy 38-year-old male accountant presents to the clinic complaining of lower back pain. The onset of pain was insidious, but the pain intensity has started to affect his work. He recently returned to his hobby, which is weight lifting after 4 months of brake he had to take to recover from his pulled hamstring. The neurological examination is normal. He has slightly limited back extension and is complaining of increased back pain during the both legs extension test while prone. What is the best initial step in the management of this patient?

A
  1. Urgent MRI
  2. Local anesthetic and corticosteroid injection
  3. Recommend increasing the number of weights he’s been lifting
    4. Simple analgesia and targeted physiotherapy

  • This patient is most likely suffering from mechanical lower back pain secondary to the lumbar muscles deconditioning during his recent injury. Targeted physiotherapy to improve core and lumbar musculature strength is recommended before he returns to weight lifting training.
  • Muscles most commonly affected in the pathology of the lower back pain are multifidus, erector spine core, and pelvic floor muscles.
  • Increased back pain on passive leg extension and gently traction with patient prone is the most sensitive test to detect lumbar spine micro-instability secondary to core and lumbar spine musculature’s weakness/deconditioning.
  • Increasing weight lifting regime prior to the core and lumbar spine musculature strengthening may lead to further deterioration of symptoms by altered muscle patterning and lack of the full stability of the spinal column.
How well did you know this?
1
Not at all
2
3
4
5
Perfectly
18
Q

A 71-year-old woman with a past medical history significant for hypertension, osteoarthritis of the spine, hyperlipidemia, opioid use disorder, and mechanical aortic valve on life prolonged anticoagulation presents to the office for a consultation for spinal cord stimulator implantation. The patient has a history of multiple back surgeries. She complains of low back pain that is burning in nature, persistent, moderate to severe, depending on the time of day. She denies numbness and tingling into her lower extremities. She states that her back pain is causing a significant impact on her quality of life. Her physical exam is unremarkable for any red flags; muscle strength, sensation, and deep tendon reflexes are within normal limits. For which of the following postoperative complications does the patient have the highest risk?

A
  1. Spinal epidural abscess
    2. Spinal epidural hematoma
  2. Dural puncture
  3. Spinal cord injury

  • Spinal epidural hematoma is a medical emergency requiring urgent neurosurgical intervention; the patient’s lifelong anticoagulation puts her at increased risk.
  • If it occurs, the hematoma must be decompressed urgently by neurosurgery.
  • Spinal epidural hematomas and spinal cord trauma are extremely rare following stimulator implantation.
  • Anticoagulation remains a relative contraindication to surgery. Given the patient’s history of valve replacement, a cardiologist should be consulted for the proper duration of stopping anticoagulation prior to surgery to limit the risk of a spinal epidural hematoma.
How well did you know this?
1
Not at all
2
3
4
5
Perfectly
19
Q

A 26-year-old female presents to the clinic for evaluation of amenorrhea. She denies being sexually active. She has missed her past four menstrual cycles. Her menarche was at 13, and the periods have been consistently 27-28 days apart. She complains of new headaches. Physical examination is unremarkable, and the patient is found to be of normal weight. Laboratory test reveals negative bHCG with a normal thyroid-stimulating hormone and prolactin level. MRI head showed a 2 cm sellar mass. The patient undergoes transsphenoidal surgery. The histological examination of the specimen shows sheets of differentiated non- keratinizing squamous epithelium with clear cystic fluid and crude papillae around fibrovascular cores. What is the likely mutation associated with this lesion?

A

1. BRAF
2. MEN
3. RET
4. CTNNB1

  • Craniopharyngioma is a neoplasm of Rathke pouch. These two theories correlate with the two histologic subtypes of craniopharyngiomas - adamantinomatous craniopharyngioma and papillary craniopharyngioma.
  • Papillary craniopharyngiomas are primarily seen in adults. Somatic mutation in BRAF has been associated with papillary craniopharyngioma. Craniopharyngiomas have benign histology but malignant behavior.
  • Somatic mutation in the CTNNB1 is proposed to be responsible for adamantinomatous craniopharyngioma development, which is commonly seen in children.
  • MEN and RET mutations predispose to multiple endocrine neoplasias which include parathyroid, pituitary, thyroid, and gastrointestinal tumors.
How well did you know this?
1
Not at all
2
3
4
5
Perfectly
20
Q

A 10-year-old male was brought in by his mother. The mother states that she has noticed recent changes in the behavior of her son. His school grades are dropping, and he has been complaining of tiredness over the past few weeks. Recently he started wetting the bed at night and experiences increased thirst and urination. Complete physical examination is unremarkable. Significant laboratory findings include fasting blood glucose 75 mg/dl, serum sodium 149 mEq/L, plasma osmolality 297 mOsm/liter, urine osmolality 280 mOsm/liter, serum thyroid stimulating hormone (TSH) 0.3 mIU/L, and free T4 0.2 ng/dL. Urine osmolality did not increase after the water deprivation test. Plasma osmolality after desmopressin administration is 285 mOsm/L. Magnetic resonance imaging showed a small, homogenous mass in the suprasellar region of the brain. Tumor oncoproteins were not elevated. He subsequently underwent an endoscopic biopsy of the mass. A dual population of cells comprising of large polygonal cells with vesicular nuclei, prominent nucleoli, and distinct cell borders intermixed with small cells with coarse, dense nuclei and scant cytoplasm was detected on histological examination. Malignant cells were also detected in cerebrospinal fluid (CSF) cytology. What is the most effective treatment modality for this patient?

A

1. Complete craniospinal irradiation
2. Ventricular radiotherapy only
3. Chemotherapy followed by stem cell or bone marrow transplant
4. Chemotherapy only

  • Diabetes insipidus is the most common presentation of germinoma involving the suprasellar region. The patient develops enuresis, polyuria, and polydipsia. Serum osmolality is greater than urine osmolality on laboratory evaluation. Desmopressin administration normalized the serum osmolality.
  • Abnormalities in the hypothalamic-pituitary axis cause panhypopituitarism and endocrinal insufficiency. This patient had hypothyroidism demonstrated by decreased serum TSH and free T4 levels.
  • Timely diagnosis and treatment are critical for better survival. Tumor staging with imaging and CSF cytology helps to guide the appropriate therapy. Whole ventricular radiation is used in cases of localized pure seminoma.
  • In patients with positive CSF cytology, more disseminated disease is suspected. Therefore, treatment is more aggressive with craniospinal irradiation for an effective outcome.
How well did you know this?
1
Not at all
2
3
4
5
Perfectly
21
Q

A 59-year-old woman presents to the clinic with concerns about hand and foot swelling and coarsening facial features. On clinical examination, she is noted to have a prominent supraorbital ridge, broad nose, acne, large lips, overbite, prognathism, tongue enlargement, and coarsening of facial features. Her fasting blood glucose level is 140 mg/dL. An IGF-1 level is obtained, which is high at 504 ng/mL (reference range: 117-321 ng/mL). She is being planned for medical treatment as she is surgically not fit. She has been prescribed a second-generation somatostatin analog. Two weeks later, she presented to the clinic with a fasting blood glucose of 200 mg/dL. Which of the following best describes the mechanism responsible for this patient’s increased blood glucose?

A
  1. Binding to somatostatin receptor 3 (SSTR3), leading to the suppression of insulin secretion
  2. Binding to somatostatin receptor 5 (SSTR5), leading to an increase in glucagon secretion
    3. Binding to somatostatin receptor 5 (SSTR5), leading to the suppression of insulin secretion
  3. Binding to somatostatin receptor 3 (SSTR3), leading to an increase in glucagon secretion

  • The patient in the clinical vignette most likely has acromegaly.
  • Pasireotide is a second-generation somatostatin analog used in the treatment of acromegaly.
  • Pasireotide preferentially binds to somatostatin receptor 5 and causes a suppression of insulin secretion.
  • Pasireotide has little impact on glucagon secretion.
How well did you know this?
1
Not at all
2
3
4
5
Perfectly
22
Q

A 30-year-old female presents with complaints of amenorrhea and white nipple discharge for the past three months. She also complains of headaches and visual changes. She has no past medical history and currently takes no medications. Her vital signs show an oxygen saturation of 98% on room air, respiratory rate of 14 breaths per minute, heart rate of 82 beats per minute, blood pressure 130/75 mmHg, and temperature of 98.6 F (37 C). Physical examination reveals bitemporal hemianopsia. Neuroimaging reveals a 1.8 cm pituitary mass abutting the optic chiasm. Which of the following is an appropriate treatment strategy?

A

1. Treatment with an oral dopaminergic agonist
2. Treatment with an oral dopaminergic antagonist
3. Transcranial resection of the tumor
4. Transsphenoidal resection of the tumor

  • The most likely diagnosis is a prolactinoma. Prolactinomas clinically present because of the mass effect (visual changes) of the tumor or because of hyperprolactinemia (amenorrhea, loss of libido).
  • Microprolactinomas (less than 1 cm) can present with symptoms of hyperprolactinemia or are detected incidentally on neuroimaging done for other reasons. Macroprolactinomas, on the other hand, often present with mass effects on the surrounding structures.
  • The preferred treatment for prolactinomas is medical therapy. Cabergoline and bromocriptine are two commonly used dopamine agonists.
  • Transsphenoidal surgery is preferred and is indicated for unsuccessful medical therapy to lower prolactin levels and decrease tumor size after several months of maximum dose medical treatment. It can also be considered in a female with a large prolactinoma (more than 3 cm) who wishes to become pregnant.
How well did you know this?
1
Not at all
2
3
4
5
Perfectly
22
Q

A 55-year-old female with a past medical history of hypertension and hyperlipidemia presents to the emergency department with a severe headache, which began an hour ago. The patient denies any head trauma. The patient denies headache like this in the past. On physical examination, blood pressure is 180/100 mm Hg, respiratory rate is 20/minute, pulse rate 100/minute, and oxygen saturation is 99% on ambient air. The patient is stuporous, and the neurological exam reveals mild hemiparesis. CT brain without contrast shows subarachnoid hemorrhage (SAH). What is this patient’s mortality risk?

A
  1. 20%
  2. 40%
  3. 60%
    4. 80%

  • According to the Hunt and Hess classification system for the severity of symptoms, this patient is in grade 4, stuporous with moderate to severe hemiparesis. Her mortality risk, according to this classification system, is 80%.
  • The Hunt and Hess classification system is used to classify the severity of SAH based on clinical findings and to predict outcome and mortality. There are 5 different grades ranging in the severity of symptoms, which correlate with the overall mortality of SAH. Grade 1: mild headache with slight nuchal rigidity with approximately 30% mortality. Grade 2: severe headache with full nuchal rigidity and no neurologic deficits other than a cranial nerve palsy with approximately 40% mortality. Grade 3: drowsiness or confusion with a mild focal deficit with approximately 50% mortality. Grade 4: stuporous with moderate to severe hemiparesis with approximately 80% mortality. Grade 5: coma with decerebrate posturing with approximately 90% mortality.
  • The modified Fisher classification describes the amount of blood seen on a head non-contrast computed tomography (CT) scan to predict the likelihood of developing vasospasm: Fisher 0: no blood detected. The incidence of symptomatic vasospasm: 0%. Fisher 1: thin focal or diffuse SAH. The incidence of symptomatic vasospasm: 18%. Fisher 2: thin focal or diffuse SAH with intraventricular hemorrhage. The incidence of symptomatic vasospasm: 35%. Fisher 3: thick focal or diffuse SAH with no intraventricular hemorrhage. The incidence of symptomatic vasospasm: 31%. Fisher 4: thick focal or diffuse SAH with intraventricular hemorrhage. The incidence of symptomatic vasospasm: 68%.
  • Oral nimodipine should be started to improve the outcome in patients with ruptured cerebral aneurysms.
How well did you know this?
1
Not at all
2
3
4
5
Perfectly
23
Q

A 46-year-old female presented with defective vision in the right eye since the age of 13 years. She has a history of fever and generalized rash over the body at the same time. Anterior segment examination reveals grade 2 nuclear sclerosis in the right eye. Fundus examination reveals mild temporal pallor. Visual acuity is 20/240, improving to 20/40 with -3D spherical correction. Color vision and central fields reveal no defect. Intraocular pressure is within normal limits. What will be the next best step in management?

A
  1. Best corrected glasses
  2. Phacoemulsification with foldable IOL in the right eye
  3. CT scan of the brain
    4. Humphrey filed analysis

  • The clinical scenario is suggestive of a patient having a cataract with temporal disc pallor. There is a history of fever and rash at a young age which is probably the reason for disc pallor. In this scenario, it is important to rule out any brain pathology before any intervention can be done. In this scenario, it is important to do a Humphrey field analysis followed by a CT scan to rule out any field defect and brain pathology. Hence, it is important to do an HFA in this case.
  • There is a possibility of amblyopia in this case.
  • Hence surgical intervention should be done under a guarded visual prognosis.
  • CT scan is mandatory to rule out any intracranial pathology and space-occupying lesion.
How well did you know this?
1
Not at all
2
3
4
5
Perfectly
23
Q

A 70-year-old man presents with chronic neck pain that radiates into his right arm. These symptoms have been ongoing for the past eight weeks. Examination reveals diminished right brachioradialis reflex, diminished sensation on the right thumb, and right wrist extension weakness. The Spurling test is positive on the right. Hoffman sign is negative bilaterally. The patient has been taking NSAIDs for the past four weeks and attending physical therapy for the past six weeks without any relief of his symptoms. Which of the following is the most appropriate next step in the management for this patient?

A
  1. Refer to neurosurgery
  2. Continue current treatment course
    3. Obtain an MRI of the cervical spine
  3. Refer for a cervical epidural steroid injection

  • The next best step for this patient is to obtain an MRI of the cervical spine to evaluate for a disc herniation and/or degenerative changes that may be causing neural foraminal stenosis.
  • Based on the exam findings, there is likely a right C6 nerve root compression that is causing the patient’s symptoms. C5-C6 and C6-C are the most common levels of disc degeneration.
  • Referral to a pain specialist or physiatrist for an epidural steroid injection is appropriate after the failure of conservative measures. However, referring healthcare providers should obtain an MRI first to confirm the diagnosis of a compressed nerve root or spinal cord.
  • Referral for operative treatment is not indicated unless all non- operative measures have been exhausted without improvement and/or the patient is experiencing progressive or severe cervical myelopathy.
How well did you know this?
1
Not at all
2
3
4
5
Perfectly
24
Q

A 39-year-old man suffers a fall from a horse and hits the occipital area on the ground. He lost consciousness momentarily and had one episode of vomiting. He is brought to the emergency department for evaluation. On arrival, he has a Glasgow coma scale of (GCS) 13/15 but follows instructions. An emergency CT head shows a 3 cm frontal contusion. He is admitted to the intensive care unit for observation. On the second day, he has a GCS of 15/15 and is sent to the ward. On the third day of admission, he becomes disoriented and minimally responsive. Repeat CT head shows the progression of the traumatic contusion. Which of the following is the most significant parameter governing the progression of the lesion observed in the patient?

A

1. The initial volume of the hemorrhage
2. Platelet count
3. Admission Glasgow coma scale
4. Location of hemorrhage

  • Hemorrhagic progression of traumatic cerebral contusion is observed in almost 75% of cases.
  • Multivariate analysis has shown that the initial volume of the hemorrhage is the most important parameter governing the risk of progression of the contusion.
  • It accounts for almost 30% predictability of the risk of hemorrhagic progression of the contusion in the follow-up imaging.
  • This parameter therefore can be a reliable clinical marker in prognosticating clinical outcomes among patients with traumatic brain contusions.
How well did you know this?
1
Not at all
2
3
4
5
Perfectly
24
Q

A 2-year-old female child presents to the clinic with her mother, with chief complaints of poor feeding, vomiting, and incessant crying over the last few days. The birth, immunization, and developmental histories are unremarkable. There is no significant medical history in any of the family members. The examination reveals an irritable child who tends to hit her head with her fists. On examination, there is a single bruise over the head and one on the chest. The head circumference is 54 cm. The temperature is 38.28 C (100.9 F). On ocular examination, the retina cannot be visualized. The rest of the exam is normal. A CT scan reveals diffuse hyperintensities in both the lateral ventricles. What is the most likely diagnosis?

A
  1. Meningitis
  2. Arteriovenous malformation rupture
    3. Subdural hemorrhage
  3. Tuberculous meningitis

  • This patient with poor feeding, incessant crying, vomiting, vitreous hemorrhage, and CT scan showing hyperintensities in the lateral ventricles is most likely a case of ‘Shaken baby syndrome.’ This is an example of countercoup brain injury. The injury occurs when the abuser shakes the baby violently, causing acceleration-deceleration related shearing forces that damage both the axons and cerebral vessels. The bruise on the head could suggest injury of the child against a hard object leading to deceleration injury.
  • Shaken baby syndrome often presents with subdural hemorrhage and retinal hemorrhage signs. However, it can also sometimes lead to vitreous hemorrhage that makes it difficult to appreciate the retinal hemorrhage.
  • Traumatic brain injury patients require a thorough general trauma evaluation. These patients need a detailed neurological examination and require a head CT for the initial assessment of brain injury.
  • Meningitis will also reveal signs of meningeal irritation absent in this patient. Photophobia and not vitreous hemorrhage is the expected ocular finding. Arteriovenous malformation rupture can also present with symptoms of raised headache, poor feeds, and vomiting. However, an ocular exam will not reveal a vitreous hemorrhage. Tuberculous meningitis will not present with symptoms of raised intracranial pressure acutely and will have a longer symptomatic phase.
How well did you know this?
1
Not at all
2
3
4
5
Perfectly
25
Q

A 3-year-old boy is being evaluated for a triangular shape of the anterior head. He has a preferred sleeping position. His forehead has a prominent midline forehead ridge. Computed tomography of the head shows that the frontal bone has a triangular shape with a closed metopic suture. There is anterior displacement of the coronal sutures, widening of the posterior parietal regions, pterional constriction, and flattening of the supraorbital ridges with lateral orbital hypoplasia. Hypotelorism is also present. All the other sutures are patent. During which of the following time periods did the involved suture most likely close in this patient?

A

1. Before 3 months
2. Between 6 months and 12 months
3. Between 12 months and 24 months
4. After 24 months

  • Metopic suture closure starts at 3-4 months of age and is complete at 8-9 months in most individuals. Premature closure before the third month will cause trigonocephaly.
  • The metopic suture closure starts at the nasion, proceeding superiorly, and terminates at the anterior fontanelle. Approximately 5.1% have the suture open after one year of age. By two years of age, it should have closed.
  • The metopic suture’s physiologic fusion occurs between 3 and 19 months.
  • A head computed tomographic scan with 3D reconstruction is performed to confirm the diagnosis and plan the surgery. It will show that frontal bones have a triangular shape, hypotelorism, anterior displacement of the coronal sutures, widening of the posterior parietal regions, pterional constriction, and flattening of the supraorbital ridges with lateral orbital hypoplasia.
How well did you know this?
1
Not at all
2
3
4
5
Perfectly
26
Q

A 67-year-old man with a past medical history of hypertension, alcohol use disorder, depression, gout, and benign prostatic hyperplasia is admitted to the intensive care unit with a grade 3 subarachnoid hemorrhage (modified Fischer scale). A cerebral aneurysm is identified on angiography, and endovascular coiling is done to reduce the risk of further bleeding. Seven days post-procedure, the patient reports nausea and headache. There is no dizziness on standing, capillary refill time is normal, and vitals show no new increase in the heart rate or fall in the blood pressure. A current set of labs is shown below. CT brain is negative for further findings. What is the best next step in treatment?

A

1. 3% sodium chloride solution
2. 0.9% sodium chloride solution
3. 0.9% sodium chloride with 5% dextrose solution
4. Total fluid restriction

  • This scenario describes symptomatic hyponatremia following subarachnoid hemorrhage (SAH) due to increased anti-diuretic hormone (ADH) secretion.
  • Hyponatremia following SAH is either due to increased secretion of ADH (majority) or cerebral salt wasting. If the hyponatremia is due to increased ADH, the patient is euvolemic. If the hyponatremia is due to cerebral salt wasting, there is clear evidence of hypovolemia.
  • Hyponatremia, here, is best treated with hypertonic saline (3% sodium chloride).
  • Some clinicians prefer normal isotonic saline (0.9% sodium chloride) if they feel hyponatremia is due to cerebral salt wasting alone. Notably, normal saline is avoided in patients with SIADH as it further drops the sodium level. Fluid restriction leads to hypovolemia, which aggravates the risk of cerebral vasospasm. Thereby, the total fluid restriction should be avoided in these people
How well did you know this?
1
Not at all
2
3
4
5
Perfectly
27
Q

A 65-year-old female with a past medical history significant for osteoporosis and type two diabetes presents to a clinic with back pain. She states that she slipped and fell on her back yesterday while working in the kitchen. She noticed a sharp electrical pain and was unable to get up for 15 minutes. Today, she is ambulating without gait instability but says the pain is not going away with her usual dose of ibuprofen. She denies saddle anesthesia, urinary, and fecal incontinence. On physical examination, she has midline and low back tenderness but no lower extremity weakness. Which of the following is the most rational initial imaging modality to be advised in the patient?

A

1. X-ray lumbar spine
2. CT Lumbar spine
3. MRI lumbar spine
4. Ultrasound of the lower back

  • The X-ray spine is the most rational imaging modality to be advised in an out-patient set up in a patient presenting with localized back pain and no neurological deficits following a fall incident.
  • X-ray is a readily available, cheap, and reliable imaging modality in evaluating and initial screening patients with a traumatic spinal injury.
  • It has high efficacy in outlining traumatic osseous spinal injuries such as fractures and subluxation.
  • It is a reliable tool in safely dichotomizing patients who need further imaging assessments with the help of CT and MRI imaging.
How well did you know this?
1
Not at all
2
3
4
5
Perfectly
28
Q

A 35-year-old man presents for continuous ringing in his right ear for the past few weeks. He also reports a constant decrease in hearing on the affected side and occasional headaches that have been worsening over the past few weeks. The physical examination is normal. His audiogram reveals sensorineural hearing loss in his right ear. MRI brain demonstrates a 2 cm lesion at the cerebellopontine angle that enhances with gadolinium and does not appear to have a “tail.” What is the most likely cause of this patient’s presentation?

A

1. Neoplasm of cranial nerve VIII
2. Vascular malformation
3. Endolymphatic hydrops
4. Meningioma

  • This patient presents with continuous, unilateral tinnitus and hearing loss. Sensorineural hearing loss along with a cerebellopontine angle (CPA) mass on MRI is consistent with the diagnosis of acoustic neuroma.
  • Tinnitus should be considered a clinical presentation rather than a disease. A neoplastic proliferation of the vestibulocochlear nerve (cranial nerve VIII) can give rise to tinnitus and concomitant hearing loss and vertigo.
  • Patients usually present with gradually progressive hearing loss and tinnitus. MRI brain is usually the modality of choice and will demonstrate a CPA tumor. Meningioma often demonstrates a “tail” of enhancement, denoting its origin from the meninges rather than from nervous structures.
  • Careful history with a description of tinnitus along with examination is the key to making an accurate diagnosis. Vascular causes of tinnitus are usually pulsatile in nature, while Meniere causes episodic tinnitus with hearing loss and vertigo and will not demonstrate a mass at the CPA.
How well did you know this?
1
Not at all
2
3
4
5
Perfectly
28
Q

What is the most common reason that adults develop spondylolisthesis?

A
  1. Malignancy
  2. Trauma
    3. Degenerative
  3. Arthritis

  • Spondylolisthesis can be congenital or develop during adulthood.
  • The disorder may result from physical stress to the lower spine from sports, carrying heavy items, trauma, and general wear and tear.
  • In most cases, it is the degenerative changes in the spinal components that lead to spondylolisthesis.
  • The condition carries a high morbidity and leads to prolonged disability
How well did you know this?
1
Not at all
2
3
4
5
Perfectly
29
Q

A 49-year-old male patient presents to the hospital for evaluation of hearing difficulty. The patient reports progressively increasing difficulty with hearing. He also complains of an occasional ringing sound in his left ear. Audiometric examination reveals an asymmetric sensorineural hearing loss in the left ear of mild severity. Non-contrast CT scan reveals a mass in the left cerebellopontine angle that is isodense. MRI reveals an avidly enhancing mass following contrast injection. The mass is seen eroding into the left internal auditory canal. A preliminary diagnosis of vestibular schwannoma is made. The patient wants to know the prognosis and available treatment options for his tumor. He makes it very clear that he wants a definitive solution that has minimum effect on his functionality. What is the most suitable option for this patient?

A

1. Surgery via retrosigmoid approach
2. Radiation therapy
3. Surgery via middle fossa approach
4. Surgery via translabyrinthine approach

  • The retrosigmoid approach is the most commonly used approach for the resection of cerebellopontine tumors.
  • Surgery provides a definitive cure and the choice of surgical approach is influenced by the size of the tumor, the extent of the tumor, and baseline hearing status.
  • The retrosigmoid approach provides maximal exposure for the resection of the tumor. A combined microscopic/endoscopic approach can help access the intracanalicular portion of the tumor.
  • Middle fossa and retrosigmoid approaches help to preserve hearing function, but the risk of facial nerve injury is slightly higher with the former.
How well did you know this?
1
Not at all
2
3
4
5
Perfectly
29
Q

A 35-year-old man with a history of chronic lower back pain presents to the clinic for further evaluation and treatment. The patient reports living an active lifestyle, playing football and soccer with his friends frequently. He also reports having some mild trauma to his back while playing but never had to see a clinician. On physical examination, there are scars on the right lower back. On palpation, the right lumbar muscles are tense and hypertrophic. On the range of motion testing, he is unable to bend laterally to his left side fully and that he feels the most amount of pain towards the end of the movement. According to the McKenzie classification, the patient’s condition is most appropriate to be classified as which of the following?

A
  1. Postural syndrome
  2. Derangement syndrome
    3. Dysfunction syndrome
  3. Sacroiliac joint disorder

  • The dysfunction syndrome is pain, which is caused by the mechanical deformation of structurally impaired soft tissue; this may be due to traumatic, inflammatory, or degenerative processes, causing tissue contraction, scarring, adhesion, or adaptive shortening.
  • The hallmark is a loss of movement and pain at the end range of motion. In this patient, he has limited side bending to the left, with most of the pain felt at the end, suggesting a dysfunction syndrome.
  • Successful treatment focuses on patient education and the mobilization of movement in the direction of the patient’s dysfunction/pain. The goal is to focus on tissue remodeling, which can be a prolonged process.
  • Sacroiliac joint disorders would fall under other or nonmechanical syndrome under the Mckenzie classification.
How well did you know this?
1
Not at all
2
3
4
5
Perfectly
30
Q

An ash leaf spot is most likely going to be seen in a patient with which condition?

A
  1. Syphilis
    2. Tuberous sclerosis
  2. Trisomy 21
  3. Lichen planus

  • There are several neurological disorders that present with cutaneous signs.
  • The ash leaf sign is often seen in patients with tuberous sclerosis.
  • The “ash leaf” sign is a light oblong depigmented patch.
  • Adenoma sebaceum is the most well-known skin feature of TS but it does not present until after puberty.
31
Q

A 59-year-old male has two months of worsening diplopia and a cranial nerve VI palsy on the right is identified. Imaging shows a clival mass which is heterogeneously enhancing on MRI T1 sequencing and hypointense on T1-weighted images without contrast. CT imaging shows a lytic lesion of the clivus with peripheral sclerosis and local invasion and contrast enhancement. Based on the possible differential diagnoses for a clival tumor which of the following is an important consideration prior to biopsy?

A
  1. Avoiding injury to the optic nerve
  2. Avoiding injury to the posterior inferior cerebellar artery (PICA)
  3. Ensuring adequate exposure to biopsy the adjacent petrous bone as well
    4. Planning for resection of the biopsy tract with resection of the tumor

  • A locally aggressive lytic lesion with heterogeneous contrast enhancement on MRI centered in the clivus is most consistent with a skull base chordoma.
  • Chordomas have a high recurrence rate and can seed the biopsy tract thus the biopsy tract should be included in the surgical resection.
  • Optimal treatment of chordomas consists of en bloc surgical resection with clean margins followed by high-dose, highly- conformal radiation such as proton beam radiation.
  • No chemotherapeutic agents have shown to be effective against chordomas.
31
Q

A 34-year-old man with traumatic brain edema is managed in the intensive care unit (ICU) with a ventriculotomy guided intracranial pressure (ICP) monitoring. The monitor reveals the onset of waves lasting 10 minutes with an amplitude of 50 mmHg above the baseline. After the wave dissipates, the clinician notes that the baseline intracranial pressure is set to a higher level than when the wave began. Which of the following is the next best step in managing this patient?

A
  1. Administer mannitol
  2. Place the patient in the supine position
    3. Lower down the external ventricular drain (EVD) bag for egress of more cerebrospinal fluid (CSF)
  3. Obtain a CT head

  • Lundberg A waves, also known as plateau waves, are waveforms of great concern in patients under ICP monitoring via EVD placement.
  • Lundberg A waves have a duration of 5 to 20 minutes and an amplitude of over 50 mmHg. The presence of a Lundberg A wave signals raised intracranial pressure and warrants immediate steps to relieve the intracranial pressure.
  • The advantage of ventriculostomy-aided ICP monitoring is its additional therapeutic benefits by allowing egress of CSF and promptly counteracting raised ICP.
  • Mannitol is another adjunct that can be used wherein there is no EVD placement. CT brain is to be planned only following measures to minimize the raised ICP temporarily.
32
Q

An 11-year-old girl with no pertinent medical history is brought to the emergency department with a history of headache, fever, purulent rhinorrhea. The mother mentions, “she has a bump in her forehead that is red and tender since yesterday” after an upper respiratory infection in the past week. On examination, the patient is alert but mildly diaphoretic, and breathing comfortably. Her extraocular movements are intact and there is prominent mucus visible around both nares. Her forehead is edematous, erythematous, and tender to palpation at the root of her left brow. Which of the following is the best initial therapy for this patient?

A
  1. Acetaminophen, ibuprofen, cefazolin
    2. Penicillin/vancomycin, ceftriaxone, metronidazole
  2. Piperacillin-tazobactam/cefazolin, meropenem, levofloxacin
  3. Needle aspiration

  • The rapid diagnosis for fast treatment of this condition is crucial for optimal outcome and decrease the risk of development of complications. Studies have demonstrated that the best strategy for the management of Pott puffy tumor is the combination of medical and surgical treatment to prevent further complications and improve morbidity and mortality.
  • Once the patient arrives and diagnosis is suspected, the patient should be admitted and started on broad-spectrum IV antibiotic, IV hydration, analgesia, and rapid coordination for imaging studies. Once the diagnosis is confirmed, ENT and neurosurgical (if intracranial component present) consultation should be done.
  • Broad-spectrum IV antibiotics should be started as soon as the diagnosis is suspected. Coverage should be provided for the most common pathogens, including gram-positive and anaerobes. It is important to choose antibiotics that have adequate blood-brain barrier penetration for central nervous system coverage. Choices include penicillin or vancomycin, 3rd generation cephalosporin, and metronidazole. Once the culture has a final result, guide antibiotic therapy for that specific pathogen. The length of therapy varies but is prolonged and includes 4-8 weeks of IV antibiotic therapy, including postoperative patients. Some small extradural collections are often treated with IV antibiotics, but aspiration/biopsy is highly recommended in order to obtain culture and guide antibiotic therapy.
  • Although other options, such as option 3 are possible, it is not the best management at this precise moment in the vignette. After ordering the administration of antibiotics, you could focus on administering analgesia, antipyretic medication, and hydration. It could be in combination with the administration of antibiotics. But between the two, antibiotics should come first if the patient is hemodynamically stable.
32
Q

A 65-year-old woman is found to have a C5-C6 herniated disc causing right C6 radiculopathy. After four weeks of physical therapy, no improvement is noted. A right-sided approach with an anterior cervical discectomy is sought, and fusion is made under general anesthesia with neck hyperextension. No intraoperative complications are observed. However, the patient develops hoarseness with difficulties in speech and swallowing after extubation. The tongue deviates to the left side with protrusion. What is the most appropriate management strategy for this complication?

A
  1. Wound exploration
    2. Speech and language therapy
  2. Dexamethasone
  3. Cold packs to the cervical area

  • The mainstay of treatment in Tapia syndrome is rehabilitation.
  • Early speech and language therapy involvement and establishing a swallowing rehabilitation program are crucial for a good outcome.
  • The role of corticosteroids in treating Tapia syndrome remains controversial, without any clear evidence, and should be guided by neurologists. In the absence of contraindications, early steroid therapy may reduce nerve swelling and enhance recovery based on the disease mechanism.
  • The patient presents left side injury to the hypoglossal (XII) and recurrent laryngeal branch of the vagus (X) nerve. Surgery was performed with a right-side approach and was not the etiology. Surgical exploration is not indicated.
32
Q

A 17-year-old male patient is brought to the emergency department following a motor vehicle accident. He is under the influence of alcohol. His pulse rate is 120/minute, and blood pressure is 110/60 mm Hg. In the primary survey, he is found to have a right-sided femur fracture, which is stabilized temporarily. His Glasgow coma scale score is eye-opening, verbal response, motor response (E1V1M5), and his pupils are bilaterally 2 mm in size and reacting to light. A computed tomogram of his brain is done, which shows a right frontal acute subdural hemorrhage with mild mass effect and no midline shift. An intracranial pressure monitor (ICP) bolt is inserted by the neurosurgeon, which shows the pressure as 15 mm Hg. The trauma surgeon suggests fixing the femur fracture, as it is associated with a hematoma and grossly displaced. He is taken to the operating room, where the anesthetist gives succinylcholine just before intubation. What is the likely value in the ICP monitor after this bolus dose?

A
  1. 5 mm Hg
  2. 10 mm Hg
  3. 15 mm Hg
    4. 20 mm Hg

  • Normal intracranial pressure (ICP) is between 5-15 mm Hg or 10-20 cm water. ICP often is increased with the administration of succinylcholine.
  • Pretreatment with defasciculation doses of nondepolarizing neuromuscular blockers reduces the intracranial pressure (ICP) increase in patients undergoing neurosurgery for brain tumors.
  • The clinical significance of this ICP rise is unclear in the acutely head-injured patient.
  • Lidocaine and short-acting opiates like fentanyl or remifentanil also may be used in an attempt to ameliorate the rise in ICP during induction.
33
Q

A 32-year-old man with a history of traumatic brain injury is admitted to the ICU. CT brain reveals multiple contusions with midline shift to right and he required decompressive craniectomy. He undergoes percutaneous tracheostomy in view of poor sensorium and anticipated need for prolonged airway protection. He develops a fever of 101.3 F (38.5 C). Blood, tracheal aspirate, and urine are sent for culture, and he is given IV piperacillin/tazobactam. The cultures show no growth. CSF evaluation is unremarkable. All the obvious sources of infections are ruled out, and all invasive lines and catheters are revised. Procalcitonin is within normal limits. Which of the following is the next best step in the management of this patient?

A
  1. IV vancomycin
  2. Repeat blood and CSF cultures
    3. Aggressive temperature control
  3. C-reactive protein (CRP) level

  • Traumatic brain injury is one of the causes of noninfectious fever in ICU. 4%-37% of patients with TBI experience fever. The reason for this fever is thought to be due to injury to the hypothalamus. When fever is encountered in ICU, first, the infectious causes should be ruled out as they have been in this case.
  • Aggressive temperature control is mandated in patients with traumatic brain injury as it is shown to reduce mortality in these patients.
  • Febrile patients can be treated either with antipyretics or physical cooling.
  • Procalcitonin is a biomarker of bacterial sepsis, which has a role in the de-escalation of antibiotics. It reduces the unnecessary use of antibiotics in the ICU. As this patient has a non-infectious fever, he does not need antibiotics. High procalcitonin levels may be associated with increased mortality in patients with sepsis.
34
Q

A 12-year-old boy is brought to the clinic with recurrent effort-induced suboccipital headaches and occasional ataxia. He is an otherwise healthy male with no past medical history or history of birth defects. Which of the following structures is most likely to be abnormal on brain magnetic resonance imaging (MRI) in this patient?

A

1. Cerebellar tonsils
2. Brain stem
3. Pons
4. Medulla

  • The Arnold Chiari or Chiari I malformation is the herniation of the cerebellar tonsils through the foramen magnum.
  • In patients with the Chiari I malformation, the bones of the skull base often are underdeveloped, which results in reduced volume of the posterior fossa, the volume of which is inadequate to contain the entire cerebellum, thus cerebellar tonsils are displaced into the cervical canal.
  • The posterior fossa in Chiari II malformation is even smaller than in Chiari I malformation. The CSF cisterns are poorly developed due to lack of brain cisterns’ expansion as a consequence of the in utero derivation of CSF circulation to the neural tube defect, all of which results in hindbrain structures downward herniation with subsequent compression of these structures against the foramen magnum.
  • In both type I and II, there is CSF flow obstruction by the foramen magnum crowding, and consequently, hydrocephalus and/or syringomyelia formation are possible over time.
35
Q

A 45-year-old man suffering from a muscular disorder for the past seven years is seen in the clinic. This disorder includes CAG repeat units, and his father had the same disease that began at age 45. Recently, the patient has experienced worsening depression, mood swings, dysphagia, weight loss, and chorea. He also fell on his left side two weeks ago while walking to the kitchen. A work-up from three months ago included an MRI brain. The enlargement seen on neuroimaging is a well-known sign of atrophy of one of the nuclei of the basal ganglia related to this patient’s underlying disease. Which of the following best describes the anatomical relationship of this nucleus to the ventricular system?

A

1. It forms the roof of the temporal horn and the lateral wall of the body of the lateral ventricle
2. It forms the floor of the temporal horn and the lateral wall of the anterior horn of the lateral ventricle
3. It forms the medial wall of the temporal horns of the lateral ventricle
4. It forms the lateral wall of the temporal horn of the lateral ventricle and the roof of the third ventricle

  • The caudate nucleus is a part of the basal ganglia and is important for memory.
  • It is a C-shaped part of the brain.
  • The head and body form the lateral wall of the anterior horn of the lateral ventricle.
  • The tail forms the roof of the inferior horn of the lateral ventricles.
36
Q

A 67-year-old male is brought into the emergency department after the sudden onset of right-sided paralysis 45 minutes ago at home. His initial vital signs include a blood pressure of 160/110 mmHg, a heart rate of 75 beats per minute, a respiratory rate of 12 breaths per minute, and a temperature of 98.7°F (37.1°C). He has a past medical history significant for coronary artery disease, atrial fibrillation, and hypertension. His medications include warfarin, propranolol, tomale, and atorvastatin. While being wheeled to the CT scanner, the patient suddenly loses consciousness, and a rapid response is called. His repeat vital signs include a heart rate of 43 beats per minute, a blood pressure of 170/60 mmHg, and a respiratory rate of 7 breaths per minute. What is the most likely etiology for this patient’s sudden decompensation?

A
  1. Neurogenic shock
  2. Acute myocardial infarction
    3. Increasing intracranial pressure
  3. Dissection of carotid artery

  • The Cushing response indicates increasing intracranial pressure and, towards its latter stages, includes hypertension, bradycardia, and bradypnea.
  • There is an increase in systolic blood pressure and a widened pulse pressure.
  • As the condition progresses, breathing will become shallow, and periods of apnea will occur.
  • If intracranial pressure continues to increase, brain stem activity will cease due to herniation, resulting in respiratory and cardiac arrest.
37
Q

A 77-year-old man is admitted to the hospital after a ground-level fall at home due to disorientation. His Glasgow Coma Scale (GCS) score is 13. He is managed conservatively for a small 1.7 cm right basal frontal contusion seen on a head CT. On the third day of admission, he is found lethargic with a GCS score of 11. A repeat CT scan shows progression of the contusion to 2.9 cm with a new area of hemorrhage in the left frontal basal area. Which of the following is most likely responsible for this progression?

A
  1. Coagulopathy
    2. Sulfonylurea receptor
  2. Thrombocytopenia
  3. Cortical venous thrombosis

  • The penumbra of the contusion experiences activation of two transcription factors (endothelial mechanosensitive mechanisms), which are the specificity protein 1 (transcription factor1) and nuclear factor kappa-B. The importance of these factors in the control of sulfonylurea receptor 1-transient receptor potential melastatin 4, which forms the regulatory subunit of the NCCa-ATP channel, is of interest in the damage cascade.
  • Nuclear factor causes central apoptosis, and specificity factor causes fragmentation of the vessels, thereby playing a pivotal role in the hemorrhagic progression of the traumatic contusions.
  • Progression usually occurs in the first 12 hours but may develop even in the 3–4 days after traumatic brain injury. Contusion progression occurs with a frequency of 63-70%.
  • Historically, it was postulated that hemorrhagic brain contusion progression occurs secondary to a coagulopathic event associated with traumatic brain injury.
38
Q

A 28-year-old overweight female is brought to the emergency department with a history of three episodes of grand mal tonic-clonic seizures. The last event occurred half an hour back, which lasted for one minute. On eliciting the history from the relatives, it is noted that she was normal until 11 PM last night when she started having a mild-moderate holocranial headache. She is nauseated but not having any vomiting, and there is no previous history of seizures. Her drug history reveals that she is on oral contraceptive pills. On examination, she is in the post-ictal phase with a Glasgow Coma Scale score of E3V4M5. The computed tomogram of the brain shows a small (2 ml volume) hematoma in the left temporal lobe. She is managed conservatively, and the next day magnetic resonance imaging of the brain with magnetic resonance venogram is ordered, which shows thrombotic occlusion of the vein of Labbe on the left side. Which of the following venous drainage pathways is impaired due to this thrombus?

A

1. Superficial middle cerebral vein to the transverse sinus
2. Deep middle cerebral vein to the transverse sinus
3. Superficial middle cerebral vein to the superior sagittal sinus
4. Deep middle cerebral vein to the superior sagittal sinus

  • The vein of Labbe is also known as the inferior anastomotic vein and drains the lateral surface of the temporal lobe.
  • The vein of Labbe connects the superficial middle cerebral vein to the transverse sinus.
  • Thrombosis of the vein of Labbe is quite rare and most commonly occurs on the left side. This condition most commonly presents with headaches and seizures.
  • The superficial middle cerebral vein is connected to the superior sagittal sinus via the superior anastomotic vein (or vein of Trolard).
38
Q

A 12-year-old boy was admitted to the emergency department after a motor vehicle collision. His initial Glasgow coma scale was 15, and his only complaint was a moderate to severe headache and several episodes of vomiting. CT of the head revealed a small epidural hematoma and several small cerebral contusions. Transfer to the local trauma center was arranged, and while awaiting transport, the nurse stated that the patient’s mental status had decreased. Physical examination of the patient revealed that he was becoming combative and his oxygen saturation was 88% on room air. Which of the following is preferred management?

A

1. Intubate and maintain target end-tidal carbon dioxide of 35 to 40 mmHg and pulse oximetry greater than 90%, elevate the head of the bed up to 30 degrees
2. Intubate and maintain target end-tidal carbon dioxide of 30 to 35 mmHg and pulse oximetry greater than 98%, administer analgesics and sedation
3. Apply the airway cannula, elevate the head up to 30 degrees, and administer analgesics
4. 100% oxygen facemask until the transfer, elevate the head up to 90 degrees

  • Airway adjuncts are indicated in patients not able to maintain an open airway or not able to maintain greater than 90% oxygen saturation with supplementary oxygen. Oxygenation parameters should be monitored using continuous pulse oximetry with a target of greater than 90% oxygen saturation. Ventilation should be monitored with continuous capnography with an end-tidal carbon dioxide target of 35 to 40 mmHg.
  • Following the strategy of a definitive airway, placement is recommended in the patient with a GCS of less than 9.
  • Serial neurological examinations allow for early identification of patients with elevated intracranial pressure (ICP), and subsequent implementation of primary bedside interventions to improve venous outflow and reduce metabolic demands.
  • Initial bedside approaches to increased ICP include elevate the head of the bed 30 degrees, ascertain that the cervical collar is not impeding venous outflow, and appropriate analgesics and sedation.
39
Q

A 34-year-old female presents to the labor and delivery unit and delivers a newborn female vaginally. She had no access to prenatal care. She does not have any significant medical history and takes no medications. Her infant was born with an enlarged head, and her eyes are red with excessive tearing. CT brain shows generalized intracranial calcifications. Which of the following is the strongest risk factor for the development of the newborn female’s condition?

A
  1. Consumption of unpasteurized dairy products
    2. Cleaning the cat litter box
  2. Working in a preschool
  3. Multiple sexual partners

  • In this question, a newborn female is born with hydrocephalus (enlarged head), intracranial calcifications, and chorioretinitis (redness and excessive tearing of the eyes). This triad suggests congenital toxoplasmosis.
  • Primary infection during pregnancy is considered very high risk for fetal infection. However, pregnant mothers with previous infections do not transmit the disease to their unborn babies.
  • Risk factors for congenital toxoplasmosis include pregnant mothers consuming raw meat and transmission via household cats. This often occurs while changing cat litter.
  • Eating unpasteurized dairy products is associated with an increased risk for Listeriosis. Multiple sexual partners increase the risk of developing the herpes simplex virus. Working in a preschool may increase the risk of exposure to Parvovirus B19. Pregnant women with a household cat should avoid changing the litter box to prevent the spread of the parasite Toxoplasma gondii.
40
Q

A 25-year-old male patient presents to the emergency department after a car accident with a piece of metal sticking out of his head. Imaging shows the piece of metal is piercing a nerve found at the dorsal midbrain. Which of these findings are expected when considering the location of the injury?

A

1. Diplopia with a downward gaze
2. Absent upward gaze
3. Inability to move the eye downward
4. Inability to move the eye upward and medially with unilateral weakness on the opposite side

  • The nerve that emerges from the dorsal midbrain is the trochlear nerve.
  • The trochlear nerve is the only cranial nerve to emerge posteriorly.
  • The trochlear nerve innervates the superior oblique muscle. This abducts, depresses and internally rotates the eye.
  • Patients with trochlear nerve palsy present wit diplopia and often have a tilt of the head towards the unaffected side as a way of compensating for the visual deficits.
41
Q

On examination, a neonate cannot move the right side of his face when smiling, and the right ear shows malformation. Newborn hearing screening demonstrates reduced hearing in the right ear. During which of the following gestational ages does the complete separation of the affected nerves occur?

A
  1. Weeks 1 to 2
  2. Weeks 4 to 5
  3. Weeks 10 to 11
  4. Weeks 14 to 15

  • Complete separation of the facial and acoustic nerves occurs between weeks 4 and 5 of gestation.
  • This is an important reference point for gestational disorders that affect both the facial nerve and auditory systems.
  • Abnormalities of the facial nerve can occur in combination with ear malformations.
  • This can allow the surgeon to predict the location of the facial nerve lesion.
41
Q

A newborn is noted to have a mass in the back, which looks like a sac containing fluids and tissue. You thoroughly examine the neonate. Which of the following finding would be of most concern?

A
  1. Vigorous symmetrical Moro reflex
  2. Open and flat anterior fontanelle
  3. Strong urinary stream
    4. Nystagmus

  • The neonate in this question has a meningocele or a meningomyelocele. In a meningomyelocele, there is spina bifida cystica with elements of the spinal cord in the cyst. A meningocele is similar, but there are only meninges and CSF in the cyst.
  • Nystagmus would raise a concern about brainstem dysfunction and is an abnormal finding in this neonate.
  • Nystagmus is a rhythmic, involuntary oscillation of one or both eyes. If present at birth or in the first 6 months of life is called infantile nystagmus.
  • Vigorous symmetrical Moro reflex, flat anterior fontanelle, and a strong urinary stream are reassuring findings.
42
Q

A 6-day-old baby in the nursery has progressively increasing head circumference which is now in the 95th percentile. The patient has a full fontanelle, sunset eyes, and poor sucking. He had repair of a lumbosacral myelomeningocele on day two after birth. CT showed features of communicating hydrocephalus. The cerebrospinal fluid culture was reported negative. What is the most rational approach for the management of this patient?

A
  1. Daily lumbar puncture
  2. Lumbar subarachnoid drain placement
  3. Daily anterior fontanelle taps
    4. Ventriculoperitoneal shunt

  • A patient with myelomeningocele may develop obstructive hydrocephalus. After cerebrospinal fluid cultures are negative, it is best treated with a shunt. A valve with a proximal ventricular catheter and a distal catheter is used.
  • The catheter is usually placed in the abdomen, but other locations include the heart and lungs. The distal end of the catheter can be located in any tissue with epithelial cells capable of absorbing the incoming CSF.
  • Ventriculoperitoneal shunts are used to treat hydrocephalus and shunt cerebrospinal fluid from the lateral ventricles into the peritoneum.
  • Hydrocephalus is a permanent problem and is not treated with serial lumbar punctures or fontanelle taps.
42
Q

After a difficult delivery a neonate is noted to have asymmetric Moro reflex. One of the arms is held in adduction, shows internal rotation at the shoulder, is pronated and extended at the elbow, and wrist flexion. Which nerve roots are most likely affected?

A
  1. C4-C5
    2. C5-C6
  2. C6-C7
  3. C7-C8

  • The description is classic for Erb palsy, a fairly common result of birth trauma that shows involvement of C5-C6 with resultant paralysis.
  • It is a complication seen in 0.5-4.4% of vaginal deliveries, often associated with shoulder dystocia. Some studies report an incidence of 0.8-1 case per 1000 births.
  • Spinal roots are much more susceptible to trauma due to absence of perineural and epineural sheaths.
  • 90% of Erb palsy resolves within a year.
43
Q

A 65-year-old male patient presents to his provider’s office due to unsteadiness. He tripped several times in the past month, more often on his left side. His wife also claims that there are times when he is inattentive and does not answer her call, especially when she is on his left side. He has no known medical comorbidities. On examination, vital signs are within normal limits. The cranial nerve examination reveals isochoric pupils, equally reactive to light, and extraocular muscular movements are full and equal on examination. There is decreased forehead crease on the left, weaker eye closure on the left and nasolabial fold is shallow on the left than the right on smiling. Weber exam localizes to the right, Rinne test shows air conduction (AC) is better than bone conduction (BC) on the right, and BC is better than AC on the left. There are no motor or sensory deficits noted. His cerebellar examination shows dysmetria and dysdiadochokinesia on the left. What is the most likely location of this patient’s lesion?

A
  1. Cerebellar aqueduct
  2. Ventral pons
    3. Cerebellopontine angle
  3. Clivus

  • The patient has a schwannoma of the 8th cranial nerve involving the vestibular division. As the tumor grows it puts pressure on the cochlear division causing tinnitus and hearing loss.
  • As the tumor enlarges, it can impinge on the facial nerve and cerebellar peduncles causing both peripheral facial palsy and cerebellar signs.
  • The diagnosis of an acoustic neuroma is made with a contrast magnetic resonance imaging (MRI) or a computed tomogram scan. Contrast is essential; otherwise, the non-enhanced scan can miss small tumors.
  • They cause 5 to 10% of all intracranial neoplasias in adults.
43
Q

A 25-year-old female is brought to the outpatient clinic with the complaint of one episode of focal seizures involving the right side of the body a day back. On examination, she is neurologically intact. She undergoes a magnetic resonance imaging scan of the brain, which shows a T1-hypo and T2-hyperintense lesion involving the left frontal lobe with mild mass effect and blooming on T2-star images. She undergoes craniotomy and decompression of the lesion. Loss of which of the following chromosomes on histopathological examination of the specimen is associated with increased patient survival?

A

1. 1p
2. 1q
3. 10p
4. 10q

  • The imaging features are suggestive of oligodendroglioma.
  • Oligodendrogliomas are usually associated with a combined loss of 1p and 19q chromosomes.
  • A loss of 1p or a combined loss of 1p and 19q is associated with longer survival.
  • The other chromosome losses have not been found to be associated with survival in oligodendrogliomas.
43
Q

A 50-year-old man arrives at the emergency department after a motor vehicle collision. The bystanders report that he was unresponsive for about 15 minutes, then started to regain consciousness. He is drowsy but able to answer questions appropriately and complains of a severe headache. He takes medication regularly for hypertension and has polycystic kidney disease. He had valvular heart surgery five years ago, his family history is significant for his father with polycystic kidney disease dying of a hemorrhagic stroke, and the patient has a smoking history of 30 pack-years. His blood pressure is 130/84 mmHg in all limbs, pulse 90/min, and respiratory rate 24/min. An examination demonstrates a conscious, well-oriented man who admits to having no memory of the motor vehicle collision. A preliminary neurological examination reveals a dilated pupil in the left eye, about 3 mm larger than the right. Extraocular movements are full and equal on the right, with difficulty of medial, upward, and downward gaze on the left. There is no facial asymmetry, the facial sensation is normal, and there is no ptosis. The motor examination does not reveal any weakness, there is a mild decrease in the vibration sense of the feet, and he ambulates without difficulty. There is an upgoing big toe bilaterally on stimulation of the lateral aspect of the foot. Nuchal rigidity is also present. He seems to have labored breathing, and a first rib fracture is suspected on the right side. Chest x-ray reveals a mass lesion at the apex of the right lung. CT scan reveals hyperdensities around the circle of Willis. Laboratory investigations, including complete blood count, glucose level, liver function tests, and renal function tests, are normal. What is the most likely cause of this patient’s pupillary defect?

A
  1. Diabetic neuropathy
  2. Epidural hematoma
    3. Aneurysmal compression
  3. Pancoast tumor

  • This patient with a fixed and dilated pupil, headache, likely autosomal dominant polycystic kidney disease (ADPKD) with secondary hypertension, probable mitral valve prolapse, and basilar hyperintensities on CT most likely has developed subarachnoid hemorrhage secondary to aneurysmal rupture.
  • The external layer of the oculomotor nerve consists of parasympathetic fibers, while the internal structures consist of motor fibers to levator palpebrae superioris and extraocular muscles. The outer layer is prone to pressure effects from epidural hematomas, aneurysms, etc. The inner layer is more prone to vascular insults like vasculopathy, infarcts, or even toxins.
  • Given his history of likely ADPKD, this patient should be further evaluated for aneurysms in the circle of Willis. A surgical repair of the compressing aneurysm should relieve the pupillary defect.
  • Diabetic neuropathy is caused by injury to the microvasculature and would present with extraocular muscle palsy rather than a pupillary defect. CT scan reveals the absence of epidural hematoma and hence is not likely in this patient. The patient should be evaluated further for the apical mass in the lung. However, it is on the right side, while the pupillary defect is on the left. A Pancoast tumor would involve the entire sympathetic trunk leading to Horner syndrome, which includes a miotic pupil rather than a dilated fixed pupil.
44
Q

A 4100-gram (99th percentile) newborn male is delivered at 39 weeks gestation to a 32-year-old gravida 3 para 3 female. The patient was born via normal vaginal delivery, but the second stage of labor was protracted. The baby is examined several hours after delivery, and there is a swollen deformity of the baby’s scalp. The patient is hemodynamically stable and well-appearing. Scalp examination revealed a soft palpable swelling over the right temporoparietal area, has slight discoloration and did not extend past the suture lines. The patient’s physical examination is otherwise unremarkable. His head circumference was noted to be 36 cm (90th percentile), and this finding remained stable throughout the patient’s hospital course. At four weeks of age, the patient presented to the emergency department with decreased feeding and a fever of 38.9 C (102 F). The only clinical symptom apart from the fever was redness and tenderness over the swelling noted during the patient’s nursery stay. Laboratory tests revealed leukocytosis, hemoglobin level of 8.8 g/L, normal platelet count, and a C-reactive protein level above 25 mg/L. Contrast enhancement was seen within the scalp lesion following the CT brain scan. Before initiating broad-spectrum antimicrobial therapy, which of the following best describes the next step in the management of the child?

A
  1. Provide reassurance
    2. Perform a needle aspiration and culture
  2. Admit patient and provide 10 days of antimicrobial treatment 4. Order serial hematocrit checks

  • Whenever an infant develops a decline in general well-being or fever, bacterial sepsis should be suspected.
  • This patient has cephalohematoma. Usually, it is a benign condition that resolves spontaneously, but rarely certain complications may occur. Due to the delivery circumstances and the clinical features noted on this patient’s presentation, infection of the cephalohematoma should be considered.
  • In addition to further imaging (to characterize the deformity as an abscess and rule out underlying osteomyelitis) as well as broad-spectrum antimicrobial therapy, in order to definitely determine an infected cephalohematoma as the source of infection, the next best step in management is to refer the patient to pediatric surgery to perform a needle aspiration and culture.
  • Due to the potential for sepsis, reassurance is not appropriate. Admitting the patient to the wards is appropriate, but the clinician should definitively diagnose the condition via needle aspiration and identify the bacterial organism before providing a prolonged course of antimicrobials. Serial hematocrits are necessary if there is a concern for ongoing hemorrhage, but the patient’s brain imaging did not indicate this.
44
Q

A 17-year-old high school basketball player has his left forearm impacted after falling and having an opponent landing on him. He complains of pain over the dorsal lateral aspect of the proximal left forearm. He then develops progressive weakness with wrist extension but denies pain or tingling in his wrist or hand. His physical examination is remarkable for tenderness over the proximal supinator muscle with 4/5 strength on wrist extension. There is also 4/5 strength with forearm supination. He has the normal sensation to light touch over radial and median distributions. Radiographs of the left forearm and wrist reveal no fractures or dislocation. Which of the following is the most likely diagnosis?

A
  1. Anterior interosseous nerve syndrome
  2. Wartenberg syndrome
    3. Posterior interosseous nerve syndrome
  3. Extensor digitorum tendon rupture

  • The posterior interosseous nerve (PIN) is a deep branch of the radial nerve arising at the level of the lateral epicondyle of the humerus.
  • The PIN provides purely motor innervation to the abductor pollicis longus, extensor pollicis brevis, extensor carpi radialis brevis, extensor pollicis longus, extensor indicis, extensor digitorum, extensor digiti minimi, extensor carpi ulnaris, and supinator muscles.
  • Entrapment or injury to the PIN would result in weakness of wrist, thumb and finger extension, and forearm supination.
  • Extensor digitorum rupture or rupture of other extensor tendons would result in loss of active extension of fingers. Wartenberg syndrome involves the superficial branch of radial nerve, which provides sensory innervation to the proximal dorsal aspect of the first 3 and 1/2 fingers and the dorsal aspect of the hand. Wartenberg syndrome is purely sensory, and no weakness of wrist or hand is noted. The anterior interosseous nerve is principally a motor nerve that arises from the median nerve with the syndrome presenting as weakness in a median nerve distribution.
45
Q

A 65-year-old male presents to the emergency department with a severe headache. He states it as the worst headache of his life. Neurological examination is unremarkable. A CT scan of the brain shows a hyperdense material filling the subarachnoid space. CT angiography of the brain shows an anterior communicating artery aneurysm. He undergoes a right pterional craniotomy and clipping of the aneurysm. There is an intraoperative rupture, and the procedure becomes complicated. Postoperatively he is found to have weakness of the left upper limb. He undergoes an MRI scan of the brain. Diffusion restriction of which of the following structures shows damage to the recurrent artery of Heubner as opposed to other lenticulostriate vessels?

A

1. Head of the caudate and anterior limb of the internal capsule
2. Caudate head and putamen
3. Caudate head only
4. Caudate, globus pallidus and putamen

  • The recurrent artery of Heubner (RAH), a branch of the anterior cerebral artery, supplies the inferior aspect of the anterior limb of the internal capsule and inferior part of the head of the caudate.
  • The involvement of the RAH thereby results in the disorder of diminished motivation like abulia and akinetic mutism along with hemiparesis with fasciobrachiocrural predominance.
  • The ultra-early diagnosis of the infarction in the caudate region due to the involvement of the recurrent artery of Heubner can be facilitated with magnetic resonance imaging (MRI) of the brain with special sequences such as diffusion-weighted images (DWI) and apparent diffusion coefficient (ADC).
  • On the contrary, in case of involvement of the medial lenticulostriate vessel, there is the involvement of the anterior part of the head of the caudate nucleus only. During the involvement of the lateral lenticulostriate vessel, there is an involvement of the caudate, anterior limb of the internal capsule as well as the putamen.
46
Q

A 60-year-old female with a history of lower extremity weakness is undergoing spine surgery using motor evoked potential monitoring. Induction of anesthesia with propofol, fentanyl, and succinylcholine was uneventful. Maintenance of anesthesia is performed with propofol and remifentanil. After verifying the full return of muscle strength following succinylcholine administration, - the neurophysiologist notes that the amplitude of baseline motor- evoked potential signals is weak. Which of the following medications could be added to increase the amplitude of the motor-evoked potentials?

A
  1. Dexmedetomidine
  2. Nitrous oxide
  3. Sevoflurane
    4. Ketamine

  • Motor-evoked potentials are a form of neurophysiologic monitoring used to evaluate the integrity of the motor pathways during surgery. Motor-evoked potentials are most commonly utilized during surgeries involving the brain or spinal cord.
  • Motor-evoked potentials are very sensitive to physiologic and pharmacologic changes, including changes in anesthetic depth, hypotension, hypoxia, and hypothermia. During MEP monitoring, it is important to maintain a steady state of anesthesia and avoid physiologic derangements that may impact motor evoked potential signals.
  • The addition of ketamine can amplify weak baseline motor- evoked potentials. Halogenated inhaled anesthetics and nitrous oxide reduce motor-evoked potentials. Propofol, benzodiazepines, and opioids reduce motor evoked potentials, although to a lesser extent than inhaled anesthetics. Dexmedetomidine and opioids have minimal impact on motor evoked potentials.
  • Intraoperative changes in MEP amplitude or latency could indicate damage to the motor pathways and must trigger discussion between the surgeon, anesthesia provider, and neurophysiologist to determine the source of change.
47
Q

A 36-year-old woman presents to the clinic for evaluation regarding chronic cluster headaches. She reports that her headaches are frequently associated with rhinorrhea, conjunctival injection, and epiphora. Input from which of the following anatomic structures is most likely contributing to this patient’s symptoms?

A
  1. Maxillary branch of the trigeminal nerve
  2. Ophthalmic branch of the trigeminal nerve
    3. Parasympathetic fibers and second-order neurons within the pterygopalatine fossa
  3. Optic nerve

  • Parasympathetic innervation through the sphenopalatine ganglia originates in the superior salivatory nucleus in the pons. Parasympathetic fibers run in the nervus intermedius, a branch of the facial nerve, through the geniculate ganglion to form the greater petrosal nerve (GPN). Parasympathetic fibers synapse in the SPG. Second-order neurons then provide secretomotor function to the nasal, oral, and pharyngeal mucous membranes, the lacrimal glands, and branches to the meningeal and cerebral blood vessels.
  • The sphenopalatine ganglion also provides sensory innervation to the nasal membranes, soft palate, and parts of the pharynx via the maxillary branch of the trigeminal nerve.
  • Sphenopalatine ganglion blocks may be considered in the setting of acute, intractable, cluster headache.
  • Because of autonomic input from the sphenopalatine ganglia, blockade of the ganglion may be associated with transient nausea and dizziness. This should be communicated to the patient while obtaining informed consent.
48
Q

A 40-year-old male without a significant past medical history presents with a severe headache that started about 14 hours ago. The patient was asleep when his headache started. The patient denies any head trauma and a family history of stroke or sudden death. CT brain without contrast is negative. On physical examination, blood pressure is 200/115 mm Hg, respiratory rate is 25/minute, pulse rate 100/minute, and oxygen saturation is 99% on ambient air. The patient is awake, alert, oriented, and can move all extremities. Which of the following is the best next step in management?

A
  1. CT angiography head and neck
  2. Magnetic resonance angiography head and neck (MRA)
    3. Lumbar puncture
  3. Digital subtraction angiography

  • To rule out subarachnoid hemorrhage, lumbar puncture is the next best step to measure the cell counts and to look for xanthochromia of cerebrospinal fluid (CSF). CSF xanthochromia due to hemoglobin breakdown products may take 4 hours to develop, but it is almost 100% sensitive between 12 hours and one week.
  • A head CT scan without contrast is the initial test for a suspected ruptured cerebral aneurysm causing subarachnoid hemorrhage (SAH). It is 100% sensitive if it is done within 6 hours of symptoms onset, but sensitivity decreases over time to 95% in 12 hours, 92% in 24 hours, and 50% in one week.
  • Once the diagnosis of SAH is confirmed, the source of bleeding can be identified with either CT angiography, MRA, or digital subtraction angiography (DSA).
  • DSA is considered the gold standard for the diagnosis of SAH due to cerebral aneurysm rupture.
48
Q

A 30-year-old male boxer presents to the emergency department for recurrent episodes of clear, watery, unilateral otorrhea. He noted the otorrhea after one of his recent fights, in which he was knocked unconscious from a blow to his right ear. It seems to worsen when he bends over to tie his shoe, and better when he turns his head to the left. He denies any fevers, chills, otalgia, tinnitus, vertigo, congestion, or nasal purulence. What is the best initial step in the management of this patient?

A
  1. Bedrest and placement of a lumbar drain
    2. Avoid straining, stooping, and blowing the nose
  2. Surgical repair
  3. Ciprofloxacin otic drops

  • Conservative management with the head end of bed elevation, bed rest, and avoidance of straining would be the correct answer choice in the case of cerebrospinal fluid (CSF) otorrhea, highlighted in the question above. The majority of temporal bone fractures and especially CSF leaks will heal on their own.
  • The precautions provided, among others, help to decrease intracranial pressure and allow for the preservation of CSF intracranially, as well as preventing further damage to the leak site.
  • Workup with beta-transferrin, determining fluid dextrose concentration, or clinical halo sign would all be appropriate to identify a CSF leak.
  • Oral amoxicillin would not be appropriate in this patient due to his lack of symptoms of acute otitis media. The patient in question is having episodes of cerebrospinal fluid (CSF) otorrhea likely from an ipsilateral temporal bone fracture. This can initially be treated with conservative measures before further surgical evaluation is warranted. CSF can be identified with the pathognomonic positive halo sign when mixed with blood. Surgical repair would not be appropriate in this patient unless he failed conservative measures. The majority of CSF leaks will heal on there own over time and with conservative treatment. Evaluation of the clear CSF with a glucose stick can be beneficial, as CSF has higher glucose concentration than either mucus or purulent drainage. Ciprofloxacin otic would not be appropriate in this patient due to his lack of acute otitis externa. The patient in question had a recent temporal bone fracture with clear otorrhea, which is likely cerebrospinal fluid (CSF). Work up with either a dextrose stick or the clinical “halo sign” would help identify the clear fluid as CSF. Most CSF leaks resolve spontaneously or with conservative measures to decrease intracranial pressure
49
Q

A 65-year-old female presents with weakness on her left side that has been ongoing for the past week. The patient is unable to get an MRI scan due to an old pacemaker. On physical examination, the clinician performs a reflex to determine if the etiology is central or peripheral. The clinician taps the distal radius to perform the reflex. Which nerve innervates the muscle used to elicit this reflex?

A

1. Radial nerve
2. Ulnar nerve
3. Median nerve
4. Posterior interosseous nerve

  • The brachioradialis muscle is innervated by the radial nerve. The clinician taps the brachioradialis muscle to elicit the Bekhterev-Jacobsohn reflex.
  • A positive or abnormal response on the Bekhterev-Jacobsohn reflex test is the abduction of the wrist and definite flexion of the fingers.
  • The Bekhterev-Jacobsohn reflex can be used in association with the Hoffman sign in the upper extremity to determine if there is an upper motor neuron lesion.
  • The brachioradialis is the muscle that is used to elicit the Bekhterev-Jacobsohn reflex. The ulnar nerve innervates the flexor muscles of the forearm and hand except for the brachioradialis.
50
Q

A 29-year-old female presents to the outpatient department with a complaint of amenorrhea for the past two months. A urine pregnancy test is negative. After gynecological evaluation, she undergoes a magnetic resonance imaging of the brain, which shows a pituitary macroadenoma with para sellar extension bilaterally. Her prolactin level is 510 ng/ml. She is started on bromocriptine, and her menstruation returns two months after initiating therapy. Of the following, which best describes the mechanism of action by which bromocriptine achieves this therapeutic effect in this patient?

A
  1. Stimulates the release of luteinizing hormone
  2. By acting as a dopamine D2 receptor antagonist
    3. Inhibiting prolactin release
  3. Acting as an estrogen receptor antagonist

  • One of the pituitary causes of amenorrhea is hyperprolactinemia.
  • As a dopamine D2 receptor agonist, bromocriptine inhibits prolactin release.
  • It is also an agonist at various serotonin receptors and inhibits glutamate release.
  • Bromocriptine is used in the treatment of prolactinoma, Parkinson disease, and type 2 diabetes.
51
Q

A 64-year-old woman is found collapsed near her bed in her nursing home after the staff outside hear a noise. On arrival of the paramedics, she has a GCS of 6 (E1V1M4), which quickly improves to 13 (E3V4M6) while transferring her to the ambulance. On arrival at the emergency department, she undergoes a CT Head, which reveals a hyperdense lesion in the right thalamus extending into the temporal lobe without midline shift. As she is being transferred from the CT scan room, her GCS drops again to 6 (E1V1M4). Her bedside observations reveal a blood pressure of 134/65 mmHg, heart rate of 74/min, and respiratory rate of 17/min. Her pulse oximetry shows an oxygen saturation of 97% on room air, and her pupils remain equal in size. Which of the following is the next best step in the management of this patient?

A
  1. An IV bolus of 500 mL 0.9% saline
  2. An IV bolus of 100 ml of 25% dextrose
  3. Administration of 100% oxygen at 6 liters per minute
    4. Lorazepam 4 mg IV

  • Patients with intracranial pathology often complain of pain. However, acute pain does not cause a decrease in the conscious level and more frequently presents with tachycardia.
  • Seizures are a common presentation for patients with acute intracranial pathology and present with an acute decline in GCS with or without convulsion. Seizures may terminate spontaneously but are often treated with anticonvulsant medication in the emergency setting. In this case, the cause of a drop in GCS could be a subclinical seizure, for which Lorazepam or Diazepam should be administered as the initial drug. Another cause of the drop in GCS is an expanding hematoma, but there are no signs to suggest the same in the scenario given.
  • In a patient diagnosed with intracranial hemorrhage, treating a systolic blood pressure of less than 140 mmHg may reduce cerebral perfusion pressure, especially if their intracranial pressure is raised.
  • Quick administration of fluid in patients who have a normal circulatory function and no acute blood loss may cause sudden spikes in blood pressure, which may increase the risk of a rebleed.
52
Q

A 5-year-old child is brought to the clinic with the complaint of diplopia. On examination, he has right lateral rectus palsy and right-sided lower motor neuron type of facial palsy. Computed tomogram shows enlarged pons with mild hydrocephalus. What is the most likely diagnosis?

A
  1. Arnold Chiari malformation type 1
  2. Medulloblastoma
  3. Ependymoma
    4. Intrinsic pontine glioma

  • The patient is having a clinical and radiological diagnosis of intrinsic pontine glioma.
  • Cranial nerves VI and VII arise from the pons.
  • Intrinsic pontine glioma is usually a radiological diagnosis.
  • The prognosis is poor for this type of lesions.
53
Q

A 65-year-old man is diagnosed with subarachnoid hemorrhage due to the rupture of an intracranial aneurysm. The aneurysm was located in the left side of the terminal branch of the vertebrobasilar artery. Which of the following branch of this artery supply the brainstem, thalamus, oculomotor and trochlear nuclei?

A
  1. P1: Pre-communicating
    2. P2: Post-communicating
  2. P3: Quadrigeminal
  3. P4: Calcarine

  • An aneurysm is a focal ballooning of the arterial vessel due to weakness within its wall. Rupture of an intracranial aneurysm is responsible for approximately 80% of subarachnoid hemorrhage.
  • The posterior cerebral arteries (PCAs) are the terminal branches of the vertebrobasilar artery.
  • From the basilar bifurcation to the junction with the posterior communicating artery, the proximal segment (P1) of PCA gives numbers of important perforating branches that supply the brainstem, thalamus, oculomotor and trochlear nuclei.
  • P2 courses between the junction and the posterior aspect of the midbrain where it gives thalamoperforating, thalamogeniculate, peduncular perforating, posterior choroidal, and posterior temporal arteries. The distal part of PCA (P3 and P4) extends from the quadrigeminal plate to the calcarine fissure, where they supply the occipital lobe, part of the parietal and temporal lobe and the posterior third of the medial brain hemisphere.
54
Q

A 42-year-old man presents to the clinic with pain radiating down his right leg and numbness over the dorsum of his right foot. He has no back pain. These symptoms began after helping his friend move three months ago. He also had a successful L4/5 microdiscectomy three years ago. On exam, there is weakness to dorsiflexion of ankle and extension of the hallux. MRI with contrast demonstrates a right-sided paracentral herniation of the L4/5 disc. There is no instability on plain radiographs. The patient has failed conservative management. Which of the following is the next best step in the management of this patient?

A
  1. L4/5 anterior lumbar interbody fusion
    2. L4/5 microdiscectomy through a midline approach
  2. L4/5 microdiscectomy through a far lateral approach
  3. L4/5 decompression with transforaminal lumbar interbody fusion

  • This clinical presentation is consistent with a recurrent lumbar disc herniation.
  • A recurrent herniation can be treated the same way as any other disc herniation with no evidence of instability or significant back pain.
  • If conservative measures fail, revision microdiscectomy is the most appropriate treatment.
  • A midline approach is the most appropriate for a paracentral disc herniation.
55
Q

A 2-year-old male was recently diagnosed with medulloblastoma. Histologic evaluation revealed sheets of small round cells, possessing a high nuclear-to-cytoplasmic ratio occasionally with neuroblastic differentiation. Although he has a rare form of medulloblastoma, the provider reports with confidence that he has a good prognosis and can have a 5-year survival of over 90% with this subgroup of medulloblastoma. What key protein undergoes constitutive activation leading to this subgroup of medulloblastoma?

A
  1. c-Myc
    2. Beta-catenin
  2. Survivin
  3. Adenomatous polyposis coli

  • In healthy cells, the canonical pathway remains inactive due to the absence of WNT, which results in low cytoplasmic concentrations of beta-catenin.
  • The pathway is activated through the binding of WNT to the Frizzled (FZD) receptor protein on the neuronal cell surface. FZD then phosphorylates Disheveled (DSH), which in turn inactivates the multimeric protein complex responsible for beta- catenin degradation.
  • Normally, a multi-protein complex comprised of axin, adenomatous polyposis coli, casein kinase 1a, and glycogen synthase kinase 3 (GSK3) binds and phosphorylates beta- catenin, subsequently promoting polyubiquitination and complete proteolysis. A second mechanism uses a different protein complex with presenilin 1 to achieve the same phosphorylation-dependent ß-catenin degradation.
  • Medulloblastoma-wingless (WNT-MB) cases demonstrate somatic mutations in CTNNB1 (86%), the gene encoding beta- catenin on chromosome 6. Mutations cluster on serine residues that serve as phosphorylation sites for beta-catenin degradation (S33 and S37). Thus, mutation at these key sites leads to constitutive activation of beta-catenin, facilitating neoplastic transformation.
56
Q

A 29-year-old female presents to the clinic with thoracic back pain. Her past medical history is unremarkable, and there has been no trauma, heavy lifting, or repetitive work maneuvers. The pain is sharp and stabbing and worse with motion or bending to unload the car after shopping. There are no other focal neurologic complaints, palpable spine tenderness, or focal neurological signs. What is a possible related clinical breast finding that may be associated with the pain?

A
  1. Palpable breast mass
  2. BRCA genetic family history
    3. Large breasts bilaterally
  3. Increased breast density on a mammogram

  • In an otherwise healthy female without risk factors for primary spine disease or breast malignancy, a prevalent cause of thoracic spine pain is large pendulous breasts with poor breast support utilization.
  • If additional history and exam confirm this, better breast support would be an essential initial step in management.
  • While it is important to determine breast cancer risk factors in any clinical presentation, which may portend risk for advanced breast cancer, breast cancer gene (BRCA) family history positivity necessitates the need for primary breast cancer screening intensification. Still, it is less likely to offer a clue for the back pain itself as metastatic malignancy is a less likely scenario in this case.
  • Breast imaging is premature in the absence of clinical breast cancer risk, and ultrasound would be a better first choice (followed by mammogram) in imaging as an initial modality in a 20’s year-old if screening were felt to be indicated.
57
Q

A 35-year-old female with a past medical history of hypertension presents to the clinic with a severe headache. She claims that she has a history of migraines; however, for the past three months, she has had a persistent right frontal headache that would decrease in intensity on taking painkillers but would not completely resolve. The headaches are more prominent upon waking up in the morning and are usually accompanied by nausea. There is no photophobia, and the patient can still do daily activities despite the headache. Her vital signs include a blood pressure of 150/80 mmHg, a heart rate of 56/min, and a temperature is 38 C. On examination, pupils were isochoric, equally reactive to light with full extraocular muscle movement. The funduscopic exam reveals papilledema bilaterally. A mild facial droop is noticed on the left when the patient is asked to smile. On motor examination, there is a mild drift of the left upper extremity with no sensory deficits. A contrast CT scan of the head was requested, which showed a homogenous, well- demarcated contrast-enhancing lesion on the right frontal lobe. Which of the following entity is the first to be involved following herniation syndrome in the patient?

A
  1. Cerebellar vermis
  2. Cerebellar tonsils
    3. Cingulate gyrus
  3. Uncus

  • The most common type of intracranial herniation is subfalcine, causing a mass effect on the frontal lobe due to a space- occupying lesion.
  • Subfalcine hernias may present with very subtle clinical symptoms. The hernia may be missed because the symptoms do not warrant neural imaging.
  • When patients develop a subfalcine hernia, the cingulate gyrus is forced under the falx cerebri, and decreased blood flow caused by compression of the ipsilateral anterior cerebral artery (ACA) results in contralateral leg weakness.
  • Subfalcine hernias may present with subtle clinical symptoms that progress to uncal or central herniation syndromes.
58
Q

A 16-year-old male with bilateral vestibular schwannomas presented with a history of progressive left-sided weakness and paresthesias for the past month. On physical examination, there was decreased sensation and hypertonia in the left upper and lower limbs, with power being 3/5. Bilateral sustained clonus and Babinski’s sign were also noted otherwise cranial nerve examination was normal. The parents mentioned a positive family history of first degree relatives with vestibular schwannomas. Which tumor will most likely be present in this patient?

A
  1. Acoustic neuroma
  2. Optic glioma
  3. Subendymal giant cell astrocytoma
    4. A dural based intracranial tumor

  • Neurofibromatosis type II is one of the neurocutaneous syndromes usually associated with multiple meningiomas and acoustic neuroma so-known as vestibular schwannoma.
  • Meningiomas are primary extra-axial dural based tumors that arise from the meninges of the brain or spinal cord and commonly seen in Neurofibromatosis type II. Optic gliomas, on the other hand, is usually seen in patients with neurofibromatosis type I.
  • Subependymal giant cell astrocytoma is usually seen in patients with tuberous sclerosis but unlike meningioma, it is not a dural based lesion. It usually arises from the brain ventricles which might cause symptoms of obstructive hydrocephalus. Whereas optic glioma can lead to gradual vision loss and proptosis of the eye.
  • Usually, meningiomas are benign, and their presence depends on their location in the central nervous system. Upper motor neuron signs are typically seen in intracranial meningiomas.
59
Q

A 3-year-old girl with a past medical history of patent foramen ovale recently is scheduled for cardiac surgery. As part of fast track anesthesia, an intrathecal morphine injection is performed for multimodal pain control. The following day, the child has diminished motor function in her right lower side. At which of the following levels was the spinal needle most likely inserted?

A

1. L2-L3
2. L3-L4
3. L4-L5
4. L5-S1

  • The spinal cord ends at L3 in newborns and migrates upward throughout childhood.
  • One of the complications from needle insertion directly into a nerve root or the spinal cord is decreased motor function which may persist long-term.
  • Pain or paresthesia during spinal will warn the anesthesiologist to withdraw the needle slightly, which may minimize the likelihood of nerve damage.
  • In the pediatric population, spinal anesthesia is typically performed after the child is already anesthetized.
60
Q

A 77-year-old woman with a past medical history of Alzheimer disease with mild cognitive impairment presents to the clinic with her son for new-onset headaches and confusion. She was recently started on a new medication classified as a monoclonal antibody that targets amyloid-beta plaques in the brain. The clinician expects an adverse effect from therapy. A brain MRI is ordered and confirms the diagnosis. What is the most likely cause of her presenting symptoms?

A
  1. Brain hemorrhage
  2. Subdural hematoma
    3. Amyloid-related imaging abnormalities-edema
  3. Encephalitis

  • Aducanumab is a newly approved agent for treating patients with Alzheimer disease (AD) with mild cognitive impairment or mild dementia.
  • The reported adverse effects are as follows: amyloid related imaging abnormalities-edema (ARIA-E) (35%); ARIA- hemosiderin deposition (ARIA-H) microhemorrhage (19%); ARIA-H superficial siderosis (15%); headache (21%); fall (15%); diarrhea (9%); confusion, delirium, altered mental status, disorientation (8%); hypersensitivity (angioedema, urticaria) (1%); immunogenicity (1%).
  • ARIA-E can be visualized as brain edema or sulcal effusions, and ARIA-H can be observed as microhemorrhage and superficial siderosis on brain imaging.
  • During clinical studies, clinical manifestations of ARIA were present in 24% of subjects who demonstrated radiographic ARIA. Symptoms included headache, confusion, delirium, altered mental status, disorientation, dizziness, vision abnormality, and nausea, the most common being headache (13%). ARIA predominantly occurred during the initial titration of therapy of the first eight doses.
61
Q

A 40-year-old man undergoes a diagnostic lumbar facet joint injection for chronic low back pain. After identifying the correct facet joint on fluoroscopy, the physician begins to inject a combination of local anesthetic and corticosteroid. During the injection, the physician encounters resistance after injecting a total of 1 mL. Which of the following is the next most appropriate course of action?

A
  1. Continue to inject the medication until a total of 1.5 mL is given
  2. Continue to inject the medication until a total of 2 mL is given
  3. Aspirate the injected medication and abort the procedure
    4. Do not inject any additional medication into the facet joint

  • The diagnostic lumbar facet joint injection should be terminated if resistance is encountered.
  • Facet capsule rupture is a potential complication of facet joint injection.
  • It is estimated that the joint space has a capacity of 1-2 mL.
  • Once intraarticular access is confirmed, a combination of local anesthetic and corticosteroid may be injected. It is recommended that the volume injected be between 1-1.5 mL, as larger volumes may rupture the joint capsule.
62
Q

A 12-year-old female presents to a clinic with a 4- month history of worsening intermittent headaches associated with dizziness. MRI shows a small mass occupying the third ventricle with areas of calcification, isointense on T1-weighted images, hyperintense on T2-weighted images. Which of the following features supports the most likely diagnosis?

A

1. Restricted diffusion on diffusion-weighted MRI
2. Disseminated disease at diagnosis
3. Older age of presentation
4. History of type 1 multiple endocrine neoplasias

  • This patient has an intracranial ependymoma, a group of glial tumors that are derived from the radial glial cells. These tumors often show restricted diffusion on diffusion-weighted images.
  • In children, older age at presentation is associated with a better prognosis. However, intracranial ependymomas are associated with poor long-term survival.
  • Anaplastic ependymomas may seed to other parts of the CNS.
  • Surgical resection may provide effective palliative therapy. After completion of therapy, an MRI of the brain is recommended every three to four months for three years.
63
Q

A 47-year-old man is scheduled for elective laparoscopic cholecystectomy under general anesthesia. Her past surgical history is significant for transsphenoidal hypophysectomy for acromegaly. Which one of the following hormones is most appropriate to be given to this patient in the perioperative period?

A
  1. TSH
  2. ACTH
    3. Cortisol
  3. Vasopressin

  • A patient who has had transsphenoidal hypophysectomy for acromegaly is likely to be lacking all of the hormones of the anterior pituitary (GH, ACTH, TSH, FSH, LH, prolactin).
  • The patient probably has been maintained on a glucocorticoid and thyroxine, both of which should be given in the perioperative period.
  • Cortisol has several roles within the body, including stimulation of gluconeogenesis, catecholamine production, and activation of antistress and anti-inflammatory pathways.
  • The posterior pituitary is usually not removed during transsphenoidal hypophysectomy; therefore, the patient is unlikely to require vasopressin therapy.
64
Q

A 65-year-old male presents with weakness in his right hand. He is asked to hold a piece of paper between the thumb and index finger with both hands. As the paper is pulled away, the patient can hold the paper in place with his left hand, but in his right hand, he has to flex the interphalangeal joint of the thumb to maintain his grip on the paper. What is the most likely diagnosis for this patient?

A
  1. Thoracic outlet impingement
    2. Compressive neuropathy in the elbow
  2. C5 radiculopathy
  3. Tenosynovitis in the metacarpophalangeal joint

  • Froment’s sign tests for palsy of the ulnar nerve. It is performed to see the action of adductor pollicis, which is the muscle of the thumb.
  • With palsy of the ulnar nerve, the individual will not be able to hold an object by adducting the thumb against the index finger but instead will have to flex the interphalangeal joint to maintain his grip on the paper.
  • Decreased action of adductor pollicis in an ulnar nerve palsy leads to the development of Froment’s sign.
  • The flexion of the thumb interphalangeal joint is controlled by flexor pollicis longus, which is innervated by the anterior interosseous branch of the median nerve.
65
Q

A 33-year-old woman is scheduled for elective surgery. The patient is already prepped and draped, and the surgeon has the scalpel in his hand. He is anxious to complete the surgery quickly since this is his fifth surgery of the day. During the time-out, the patient states that she does not want to be resuscitated if an emergency develops, but she has not documented previously this wish. The surgeon should next do which of the following?

A
  1. Say that there are witnesses so no further documentation needs to be performed
    2. Stop the procedure and have the patient document her wishes for end of life care on the corresponding state scope of treatment form
  2. Call the hospital ethics committee chairperson to serve as a witness prior to proceeding with the surgery
  3. Allow the surgery to proceed and afterwards have the patient document her wishes for end of life care on the corresponding state scope of treatment form

  • Communication failure is one of the most common causes of medical error.
  • A time-out is a key safety tool that is often required for any procedure. It involves confirmation of patient and procedure, and it allows members of the healthcare team to ask questions if they need clarification.
  • Although some states have statutes (such as Indiana) that clarify that a patient’s spoken wishes in the presence of a physician witness are adequate for defense of a physician’s treatment, other states (such as Kentucky across the Ohio River) do not. A physician should not proceed without knowing the law in the state of practice unless he/she is willing to allow the issue to be determined by a court after the fact.
  • Non-emergent procedures should be delayed in order to follow hospital or higher authority ethical and legal precautions.
66
Q

A 52-year-old male presents with a slowly growing mass of the hard palate. It is asymptomatic and measures 0.75 x 0.75 cm. No other abnormalities are found. Needle biopsy shows the most common tumor in this location. Select the best management.

A
  1. Observation
  2. Enucleation
    3. Complete excision ensuring clear margins
  3. Wide excision with lymph node dissection

  • Pleomorphic adenomas should be completely excised because there is a risk of recurrence.
  • Recurrence also can occur secondary to inoculation.
  • Lymph node dissection is not needed.
  • Malignant degeneration occurs in 10% of people who are not treated.
67
Q

A 57-year-old man presents with acute severe back pain. The pain radiated to the left leg and foot after lifting some heavy pots in his garden four days ago. He describes the pain as nine out of ten, and it keeps him up at night. The patient has been taking ibuprofen 800 mg three times a day and acetaminophen over the counter with no relief. He denies weakness, numbness, or tingling. His past medical history includes a spinal fusion of the L1-L2 vertebrae and chronic fatigue syndrome. On examination, the straight leg raise on the left is positive, reproducing his pain. His muscle strength, sensation, and deep tendon reflexes are all within normal limits. What is the most appropriate next step in the management of this patient’s pain?

A
  1. Gabapentin
  2. Cyclobenzaprine
  3. Prednisone
    4. Hydrocodone with acetaminophen

  • Acute cervical and lumbar radiculopathies due to a herniated disc are primarily managed with non-surgical treatments. NSAIDs and physical therapy are first-line treatment modalities if they fail to relieve the patient’s pain; an opioid such as hydrocodone can be prescribed for a short time. Long-term use of opioids is not recommended.
  • The straight leg raise and the absent Achilles deep tendon reflex (DTR) indicate the location of the disc herniation. Another test that can be performed would be to have the patient perform heel and toe walking; heel walking tests the motor function of the L4 nerve root, and toe walking tests the motor function of the S1 nerve root.
  • Paraspinous muscle spasm is a nonspecific finding.
  • At this point, the patient should probably be sent for an MRI, although in some cases, therapy and watchful waiting may be in order. This decision would be made based on the severity of the patient’s symptoms.
68
Q

A 16-year-old patient is rushed to the emergency department following a motor vehicle collision. He is complaining of localized neck pain without any radiation towards his upper limbs. His neurological examination did not reveal any sensory or motor deficits. All the deep tendon reflexes are normal. He has a normal single breath count and has no features suggestive of Horner syndrome. The CT of the cervical spine, however, revealed a complete spondyloptosis at the interface of the fifth and sixth cervical spine (C5-6). The patient also had fractures of the bony components of the Denis third columns. What is the preferable method of posterior implant fixation at the subluxated segments of the cervical spine in this patient?

A
  1. Translaminar screw fixation
  2. Sublaminar wiring
  3. Interspinous wiring
    4. Lateral mass screw fixation

  • The patients with complete traumatic spondyloptosis of the cervical spine sometimes can paradoxically present without any neurological deficits during the clinical examination. These patients invariably have concurrent bilateral fractures of the lamina, which are the components of the posterior element of the cervical vertebra (third Denis columns).
  • Such fractures of the lamina help to prevent compression and the subsequent damage to the spinal cord despite such a higher grade of subluxation. These fractures help to dissipate the intraspinal pressure, thereby preserving the neurological functions in the patient.
  • Lateral mass screw fixation is the preferable method of posterior implant fixation in cases of higher Meyerding grades of traumatic subluxations. The reduction of the locked facets, lateral mass screw fixation at the subluxation segments, and the translaminar screw placement in the distal normal segment is the preferable method for posterior fixation.
  • The concurrent fractures of the lamina and the spinous process will obviate the placement of the sublaminar and the interspinous wiring for anatomical stabilization of the spine. Moreover, they are only applied for in-situ fixation of the spine in patients with poor American spinal injury association (ASIA) neurological status for allowing early wheelchair mobilization of such patients.
69
Q

A 29-year-old man presents to the emergency department (ED) with a significant neck injury following a major road traffic accident (RTA). His neurological exam is within normal limits. The rest of his clinical, systemic assessment is also within normal limits. On detailed radiological evaluation (including plain radiographs, CT, and MRI), he is diagnosed with type 3 fracture of odontoid extending onto C2 body, along with C1 arch fracture and C1-2 subluxation. He is scheduled for posterior C1-2 instrumented fusion. Pre-operatively, CT angiography is performed, which revealed complete left vertebral artery disruption. Which of the following intraoperative steps is most likely to minimize the chances of a brainstem stroke in this patient?

A

1. Avoidance of right C2 pedicle screw
2. Avoidance of left C2 pedicle screw
3. Avoidance of left C2 trans-articular screw
4. Avoidance of right C2 translaminar screw

  • Vertebral artery injury is a known complication of C2 pars, pedicle, and trans-articular screw insertions.
  • Therefore, whenever there is disruption of one vertebral artery (either traumatically or iatrogenically), it is of utmost importance to preserve the contralateral vertebral artery to prevent devastating complications of brain stem ischemia.
  • In such situations, alternate C2 fixation techniques (like lamina- based translaminar screws or sublaminar wiring) can be safe options for contralateral C2 instrumentation.
  • Disruption of bilateral vertebral arteries can result in major brainstem strokes, causing neurological deficits (lateral medullary syndrome) and even death.
70
Q

A 43-year-old man presents to the healthcare provider after a motor vehicle collision. The patient is unconscious and unresponsive. An x-ray of the head confirms a fracture of the skull through the parietal foramen, while a CT scan reveals a hemorrhage. The vein that passes through the parietal foramen connects which of the following pair of veins?

A

1. Superior sagittal sinus and superficial temporal vein
2. Occipital sinus and vertebral vein
3. Transverse sinus and posterior auricular vein
4. Cavernous sinus and supraorbital vein

  • The parietal emissary vein in the parietal bone joins the superior sagittal sinus and branches of the superficial temporal vein on the scalp.
  • The emissary veins do not contain valves.
  • These veins connect the superficial veins of the upper face and scalp with the diploic veins of the skull.
  • Any infection in the face and scalp area can easily be transmitted to the brain via the emissary veins.
71
Q

A 30-year-old female is brought to the emergency department with the complaint of one episode of seizures involving the left side of the body three hours back, which lasted for 2 minutes. She was conscious at that time, and the ictus involved tonic posturing of the left upper and lower limbs. This was the first time she was having seizures in her life. She does not have any comorbidities, and she is a smoker. There is no history of trauma. On examination, she is conscious and oriented with no neurological deficits. She undergoes a computed tomogram of the brain, which shows a 2x1 cm sized hematoma in the right temporal lobe. She is managed conservatively. She undergoes magnetic resonance imaging (MRI) of the brain one month later, in which the gradient echo sequence shows a berry appearance with a rim of signal loss at the hematoma site. She is scheduled for surgery. Before that, a contrast MRI of the brain is ordered. What is its role in preoperative planning?

A
  1. To see the number of lesions
  2. To assess the size of the lesion
  3. To assess enhancing lesions accessible for embolization
    4. To assess associated developmental venous anomaly

  • Gradient echo sequences (GRE), not post-contrast sequences, are most sensitive for assessing the number of lesions.
  • Post-contrast images are, in particular, important to see any associated developmental venous anomaly.
  • Developmental venous anomaly (DVA), if damaged intraoperatively, leads to brain venous infarction postoperatively.
  • Mixed cavernous malformation refers to coexistent DVA with a cavernous angioma.
72
Q

A baby is born by C-section due to cephalopelvic disproportion. The anterior fontanelle is closed with a palpable coronal suture ridging, hypertelorism, and syndactyly of the hand and feet. What is the most correct in terms of management for this child?

A
  1. Reassurance
    2. Surgery by 6 to 12 months of age
  2. Helmet
  3. Surgery within the first month of life

  • Craniosynostosis correction should be done between 6 to 12 months of age when there are no signs of increased intracranial pressure (ICP) or airways obstruction. This correlates with the period of time in which the infant’s brain and head grow the most. The main goal of surgery is to create enough space in the cranial vault for the brain to grow and develop properly as well as to provide the child with a more acceptable appearance.
  • Left untreated craniosynostosis can affect the development of the child. This is due to restriction for growth of the brain and damage to the brain tissue due to increased ICP. This is why surgery is the most correct treatment.
  • When surgical intervention is done in a timely manner, has a very good outcome with fairly normal growth and development. Follow up of a further fusion of sutures and head growth is crucial to determine the need for re-intervention in these patients, with a special focus on syndromic craniosynostosis
  • The child should be managed by an interdisciplinary team, which includes a neurologist. Depending on the type of surgery, the need for the subsequent use of a helmet might arise but is not the mainstay treatment.
73
Q

A 50-year-old female presents to the clinic with generalized weakness and difficulty in walking for the past three to four months. She has more difficulty in climbing stairs, combing hair, and standing from the chair. She denies fever, pain, trauma, numbness, paresthesia, urinary, or bowel symptoms. On further inquiry, she reveals that she has constipation, sleeps a lot, and has gained 12 kgs of weight even though her appetite has decreased. On physical examination, her skin seems dry. Muscle strength is 3/5 in proximal muscles and 4/5 in distal muscles of both upper and lower limbs. Deep tendon reflexes are present with delayed relaxation. What is the next step in the diagnosis of this condition?

A
  1. Creatine kinase (CK)
    2. Thyroid-stimulating hormone (TSH)
  2. Electromyography
  3. Hemoglobin A1c

  • The patient is most likely having myopathy, secondary to hypothyroidism. Thyroid-stimulating hormone (TSH) should be checked to confirm the diagnosis.
  • Creatine kinase (CK) is a non-specific test and can be elevated in almost all myopathies.
  • Electromyography (EMG) may reveal features of myopathy, but it will not differentiate hypothyroid myopathy from other causes of myopathy.
  • In patients with symmetrical polyneuropathy in glove and stocking distribution, hemoglobin A1c is usually checked to look for underlying diabetes mellitus
74
Q

A 45-year-old woman is undergoing microsurgical clipping of the anterior communicating artery aneurysm. The surgeon wants to prevent complications that can occur from an inadvertent clipping of the perforators. Which of the following modalities can best help in achieving this?

A
  1. Intraoperative adenosine injection
  2. Transcranial Doppler study
    3. Indocyanine green angiography
  3. Endoscopy

  • Incomplete clipping of the aneurysm can lead to rebleed. The inadvertent clipping of the main branches or the perforators can lead to postoperative vasospasm.
  • Indocyanine green is an intraoperative, readily available, and safe vascular flow study to ensure complete occlusion of the aneurysm along with validating the patency of the branching vessels and their perforators.
  • Indocyanine green flow study gives visual confirmation of the flow within the studied vessels.
  • The endoscopy only helps in visual confirmation of clip placement at the blind surgical corridors. The transcranial Doppler study only ensures flow in the vessels and the perforators, but it does not quantity the flow through them. Hypoperfusion with decreased flow velocity predisposes to vasospasm. It is operator-dependent with high interrater bias. Adenosine injection can help in achieving a temporary cardiac arrest for the purpose of clipping.
75
Q

Wernicke syndrome, often seen in patients with chronic alcohol use disorder, typically presents with which of the following symptoms?

A

1. Visual changes, ataxic gait, confusion
2. Amnesia, hallucinations, confabulation
3. Dementia, dermatitis, diarrhea
4. Tremors, confusion, autonomic hyperactivity

  • Wernicke syndrome is due to thiamine deficiency and seen in malnourished pts like patients with chronic alcohol use disorder; they have lesions in the mamillary bodies.
  • Korsakoff psychosis consists anterograde and retrograde amnesia, confabulation and confusion.
  • Treatment is with thiamine replacement; there is little supporting evidence that treatment of comatose patients with dextrose prior to thiamine will precipitate acute Wernicke’s encephalopathy.
  • Patients with delirium tremens will present 3-5 days after last alcohol intake with confusion, agitation, tremors, hallucinations, and increased autonomic activity like tachycardia, fever, elevated BP.
76
Q

A 22-year-old male with a past medical history of high spinal cord injury undergoes laparoscopic direct diaphragmatic pacing for the persistent requirement of mechanical ventilation. On the second-day post-operation patient’s temperature is 39 degrees Celsius, heart rate is 120 beats per minute, blood pressure is 70/40 mmHg. He is complaining of generalized abdominal pain. Pertinent examination findings reveal tachycardia, decreased capillary refill, and abdominal tenderness with guarding and rigidity. Which of the following should be immediate management in this patient?

A

1. Obtain an upright radiograph and urgent surgical consultation.
2. Obtain a right upper quadrant ultrasound.
3. Urgent laparotomy for pacer lead extraction.
4. Fluids, antibiotics, and watchful waiting.

  • The patient probably has perforated viscus leading to peritonitis.
  • Perforated viscus with peritonitis is a dreaded complication of the diaphragmatic pacing procedure. The patient would usually present within hours of the surgery with the above symptoms. An upright radiograph would show air under the diaphragm.
  • Even if the upright radiograph doesn’t show air under the diaphragm, the team should have a high index of suspicion for perforated viscus. Urgent surgical consultation with optimal imaging should be done for the diagnosis.
  • The patient is in septic shock. He should receive fluid resuscitation, blood cultures, and appropriate antibiotics. However, watchful waiting is not an appropriate management strategy as the ultimate treatment of his septic shock is source control.
77
Q

A 32-year-old right-handed female with a past medical history of systemic lupus erythematosus (SLE) presents to the emergency department with nausea, dizziness, headache, confusion, vertigo, dysarthria, and bilateral central vision loss with the onset of symptoms two hours ago. On examination, the temperature is 37 C, pulse rate is 90/min, blood pressure is 165/73 mmHg, and respiratory rate is 36/min breaths per minute. The patient is alert but confused and complaining of a headache. Her blood glucose level is 93 mg/dL, and the National Institute of Health stroke scale is 7. CT scan of the head without contrast is negative for an acute bleed, and subsequent CT angiogram of the head and neck shows a large vessel occlusion (LVO) at the basilar tip. Which of the following steps is recommended at this time?

A
  1. MRI scan of the head and neck to characterize the infarct
  2. CT perfusion scan to measure the extent of the infarct
    3. Mechanical thrombectomy
  3. Antiplatelet medications

  • MRI scan of the head and neck is not indicated at this time. The patient should immediately receive a tissue plasminogen activator (tPA) such as alteplase and be transferred to an angiography suite for mechanical thrombectomy (MT).
  • There is no need to delay the treatment with unnecessary testing, and the patient should immediately receive alteplase.
  • Mechanical thrombectomy (MT) is recommended in this case based on the recent DAWN and DEFUSE-3 trials for anterior circulation.
  • Everyone should be considered for alteplase administration as per NINDS trial results published in 1993, which showed significant mortality and outcomes benefits of alteplase administration if the patient has arrived in the emergency department (ED) within the three hours window and has no contraindication to alteplase. The patient should first receive alteplase as she is within the tPA window. Although after tPA administration, the patient should be moved to an angiography suite for MT as the symptom onset is within 24 hours, and there is an LVO at the basilar tip, which would require MT.
78
Q

A 19-year-old G1P0 woman at 19 weeks gestation presents to the emergency department after being assaulted by her partner. She has diffuse ecchymosis around her right zygoma, decreased visual acuity, diplopia, and gross hyphemia. Extraocular movements are asymmetric, and the patient complains of a right-sided throbbing headache. Her GCS is 15. She has no focal deficits and did not lose consciousness. MRI is unavailable, and the patient states she does not want any imaging study that would harm her fetus. An optic nerve sheath ultrasound (ONSUS) is planned for risk stratification. Which of the following is the next best step in managing this patient?

A
  1. Proceed with ONSUS
  2. Proceed with ONSUS after 24 hours
    3. Evaluate the patient for an open globe injury prior to ONSUS
  3. Reassure the patient that there is no risk to the patient with a CT head

  • Based on the presentation, this patient is at low risk for ICH. However, her injuries are consistent with an open globe or severe ocular injury. The only absolute contraindication for ONSUS is an open globe, and an ophthalmologist should be consulted immediately.
  • Signs and symptoms of an open globe include visible eyewall defects, severe chemosis, hypotony, and total hyphema. If there is any suspicion of an open globe, avoid performing the study in the affected eye.
  • Ocular ultrasound is unlikely to expose the fetus to any harm and does not expose the mother to ionizing radiation. This study would NOT be contraindicated if there was no concern for an open globe.
  • It is worth noting again that the optic nerve sheath diameter (ONSD) has not been well validated in pregnant patients. There is evidence of several pregnancy-related conditions (i.e., preeclampsia) that affect ONSD. Use this study with caution in pregnant and pediatric patients.
79
Q

A 65-year-old man with a past medical history significant for hypertension, diabetes mellitus, and smoking is brought to the clinic by his daughter with chief concerns of memory loss. The patient has had a rapid decline in cognitive and memory function, having problems with activities of daily living and paying his bills all within the past year. MRI of the head reveals infarcts in multiple areas, including the posterior hippocampus. What vessel normally provides blood to the posterior hippocampus?

A
  1. Posterior cerebral artery
  2. Anterior choroidal artery
  3. Anterior cerebral artery
  4. Anterior inferior cerebellar artery